You are on page 1of 94

1

Valvular Heart Disease


Question 1
A 62-year-old male with history of bileaflet mechanical aortic valve replacement (AVR)
and ascending aorta replacement 4 years ago is seen in the clinic for preoperative
evaluation prior to planned open nephrectomy. He has a history of well-controlled
hypertension. He does not exercise regularly, but reports being able to climb two flights
of stairs without difficulty. His daily medications are aspirin 81 mg, amlodipine 5 mg, and
warfarin 6 mg. His examination reveals a blood pressure of 128/73 mm Hg and a heart
rate of 74 bpm. His neck veins are not distended. His heart sounds are regular with a
crisp mechanical S2 and soft systolic murmur at the right upper sternal border. His
lungs are clear, and the remainder of his examination is normal. His surgeon requests
that warfarin be discontinued 4 days prior to surgery.
In addition to stopping warfarin, which of the following would you recommend for the
perioperative period in this patient?
A. An exercise nuclear stress test.
B. An infusion of unfractionated heparin.
C. Low-molecular weight heparin.
D. A tranthoracic echocardiogram.
E. Continue aspirin 81 mg.

Per the 2017 updated to valvular heart disease guideline:


Temporary interruption of vitamin K antagonist (VKA) anticoagulation, without bridging
agents while the interantional normalized ratio (INR) is subtherapeutic, is recommended
in patients with a bileaflet mechanical AVR and no other risk factors for thrombosis who
are undergoing invasive or surgical procedures.
“Bridging” therapy with either intravenous unfractionated heparin or low-molecular-
weight heparin has evolved empirically to reduce thromboembolic events during
temporary interruption of oral anticoagulation in higher-risk patients, such as those with
a mechanical mitral valve replacement or AVR and additional risk factors for
thromboembolism (e.g., atrial fibrillation, previous thromboembolism, hypercoagulable
condition, older-generation mechanical valves [ball-cage or tilting disc], left ventricular
systolic dysfunction, or >1 mechanical valve).

KKUH
Collected by:
Dr Hani Abdullah
Dr Fatma Hadi
Dr Salem Boresa
Dr Rami Elesali
Dr Naeif Almagal
1/1/2020
2

When interruption of oral VKA therapy is deemed necessary, the agent is usually
stopped 3-4 days before the procedure (so the INR falls to 4 METs by history and so a
preoperative stress test is not needed. Absent a suspected change in valve function,
there is no need to repeat the echocardiogram at this time.
Answer E
Key Point
Periodic clinical and echocardiographic evaluation of patients with a prosthetic valve is
essential for early detection of prosthetic valve dysfunction. Careful medical
management of patients with prosthetic valves includes careful control of antithrombotic
therapy and prescription of infective endocarditis prophylaxis.

Question 2
-A 22-year-old female is referred to you for advice regarding anticoagulation
management if she gets pregnant. She had mitral valve (MV) endocarditis 3 years ago
treated with a mechanical MV replacement. She has been on warfarin since then. She
has clinically done well and is asymptomatic. She wants to have children. Her
examination reveals the mechanical MV sounds and is otherwise normal.
Her medications include birth control pills, aspirin 81 mg, and warfarin 4 mg each
evening. She has been reliably following her international normalized ratios and the
values range from 2.5-3.0 monthly.
Based on current guidelines, which of the following medical options would you
recommend to her in regard to the use of anticoagulation during pregnancy?
A. Weight-based low molecular weight heparin for the first trimester, warfarin for the
second and third trimester, and then unfractionated heparin just prior to delivery.
B. Warfarin for the entire pregnancy until the time of delivery and then a change to
unfractionated heparin.
C. Counsel against pregnancy due to the risks involved.
D. Weight-based unfractionated heparin during the entire pregnancy.
E. Weight-based low molecular weight heparin for the entire pregnancy and then a
change to unfractionated heparin just prior to delivery.

KKUH
Collected by:
Dr Hani Abdullah
Dr Fatma Hadi
Dr Salem Boresa
Dr Rami Elesali
Dr Naeif Almagal
1/1/2020
3

The correct answer is that she may continue to use her low-dose warfarin for the entire
pregnancy and change to unfractionated heparin (UHF; with an activated partial
thromboplastin time of more than twofold control) just prior to vaginal delivery. This is
because the embryopathy from warfarin appears to be dose dependent. It is now a
Class IIa (Level of Evidence B) recommendation that those patients on ≤5 mg/day of
warfarin may safely remain on warfarin the entire pregnancy, changing to UFH just prior
to delivery.
The other options may be considered if the daily dose of warfarin is >5 mg/day. In that
situation, the warfarin should be discontinued for the first trimester and either UFH (with
measured activated partial thromboplastin time of more than twofold control) or low
molecular wieght heparin (LMWH; with measured anti-Xa of 0.8-1.2 U/ml 4-6 hours
postdose) may be used. Weight-based dosing of UFH and LMWH does not provide
adequate anticoagulant effect in many cases because of the increased volume of
distribution that occurs during pregnancy. Warfarin then can be reinstituted for the
second and third trimester and stopped just prior to delivery and replaced with UFH as
mentioned earlier.
Prosthetic heart valves are not a contraindication to pregnancy.
Answer B
Key Point
Selection of valve procedure (repair vs. replacement, surgical vs. transcatheter) and
prosthetic valve type (mechanical vs. bioprosthetic) has to be individualized according
to the patient’s characteristics, including age, surgical risk, and personal preferences.

KKUH
Collected by:
Dr Hani Abdullah
Dr Fatma Hadi
Dr Salem Boresa
Dr Rami Elesali
Dr Naeif Almagal
1/1/2020
4

Question 3
3-A 30-year-old male is referred for evaluation of a murmur. He reports no symptoms
and has no significant medical history. On physical examination his height is 69 inches
and weight is 185 lbs. His blood pressure is 135/70 mm Hg with a regular heart rate of
78 bpm. His lungs are clear. Prominent carotid pulsations are present. The jugular
venous pulse is at the level of the sternal notch. The apical impulse is slightly enlarged
and laterally displaced to the anterior axillary line. The S1 and S2 are normal, and an S3
is present. There is an early systolic click that does not change with inspiration. Both a
soft (grade 2/6) crescendo-decrescendo systolic murmur and a soft (grade 2/4)
decrescendo diastolic murmur are present along the left sternal border.

Which of the following is the most likely valvular abnormality in this patient?
A. Bicuspid aortic valve with regurgitation.
B. Patent ductus arteriosus.
C. Pulmonic valve stenosis with regurgitation.
D. Rheumatic mitral stenosis and regurgitation.
E. Degenerative aortic valve stenosis with regurgitation.

Bicuspid aortic valve can be associated with sudden cessation of valve opening, leading
to an ejection systolic click, and often is associated with significant aortic valve
regurgitation. Findings of chronic aortic regurgitation with left ventricular volume
overload include an enlarged and laterally displaced apical pulse, wide aortic pulse
pressure, and an S3.
Pulmonic valve stenosis is unlikely because the murmur does not change with
respiration and the jugular venous pulse is normal. Additionally the intensity of the
pulmonic ejection click decreases with inspiration.
Degenerative aortic stenosis is not associated with a systolic click and is less likely
given his young age.
Rheumatic mitral stenosis may be associated with a diastolic click, but not one that
occurs during systole.
Patent ductus arteriosus produces a continuous machine-like murmur and is not
associated with a click or wide pulse pressure. Answer A

KKUH
Collected by:
Dr Hani Abdullah
Dr Fatma Hadi
Dr Salem Boresa
Dr Rami Elesali
Dr Naeif Almagal
1/1/2020
5

Question 4
4-A 54-year-old male presents for routine evaluation for known chronic aortic valve
regurgitation due to bicuspid aortic valve. He remains asymptomatic. His vital signs
include a heart rate of 84 bpm and blood pressure of 140/50 mm Hg. His physical
examination reveals a grade 2/6 systolic ejection murmur along the left sternal border
and a grade 3/4 diastolic flow murmur. His echocardiogram reveals severe aortic
regurgitation.

Which of the following echocardiographic parameters would be an indication for surgical


aortic valve replacement at this time?
A. A left ventricular end-diastolic dimension of 6.0 cm.
B. A vena contracta width of 0.7 cm.
C. A left ventricular end-systolic dimension of 5.3 cm.
D. A left ventricular ejection fraction of 55%.
E. An aortic regurgitant fraction of 65%.

Based on the 2014 American College of Cardiology/American Heart Association


valvular heart disease guideline, the patient has evidence for severe aortic valve
regurgitation. Aortic valve replacement (AVR) is indicated for the asymptomatic patient
with severe aortic regurgitation when there is evidence of left ventricular (LV) systolic
dysfunction (LV ejection fraction <50%) or other cardiac surgery is planned (Class I
indications; see Figure 1). AVR is also indicated if there is LVEF ≥50% but the left
ventricular end-systolic dimension (LVESD) is >50 mm (Class IIa) or left ventricular end-
diastolic dimension (LVEDD) is >65 mm and the patient is of low surgical risk (Class
IIb). The other echocardiographic parameters (vena contracta width, regurgitant
fraction) confirm severity of aortic regurgitation, but are not critical factors in the decision
for surgery. Answer C

KKUH
Collected by:
Dr Hani Abdullah
Dr Fatma Hadi
Dr Salem Boresa
Dr Rami Elesali
Dr Naeif Almagal
1/1/2020
6

Key Point
Left ventricular dysfunction and dilatation can be reversible if valve replacement is
performed in a timely fashion. Therefore, serial follow-up with clinical examinations and
echocardiography is recommended to identify patients who require intervention before
symptoms develop.

KKUH
Collected by:
Dr Hani Abdullah
Dr Fatma Hadi
Dr Salem Boresa
Dr Rami Elesali
Dr Naeif Almagal
1/1/2020
7

Question 5
5-A 33-year-old male is admitted with a 1-week history of fever and shortness of breath.
He has a history of intravenous drug use.
On examination, he has a temperature of 102.5 degrees Fahrenheit, a heart rate of 110
bpm, and a blood pressure of 102/60 mm Hg. He appears mildly dyspneic and
diaphoretic. His jugular venous pressure is 10 cm H20. His lungs have bibasilar rales.
His cardiac exam demonstrates a hyperdynamic precordium with a soft systolic murmur
and S3 gallop. His extremities are warm without edema.
His echocardiogram on admission shows a left ventricular ejection fraction of 75%, a 1
cm vegetation on the tricuspid valve with moderate to severe tricuspid regurgitation, and
thickening of the mitral valve with severe mitral regurgitation. His chest X-ray shows
pulmonary edema.
He is admitted to telemetry. Serial blood cultures are sent and intravenous vancomycin
is started.
What is the next best step in the management of this patient?

A. Intra-aortic balloon counterpulsation.


B. Urgent valve replacement.
C. Repeat transesophageal echocardiogram in 6 weeks.
D. Initiation of sacubitril/valsartan.
E. Addition of rifampin.

Early surgery (during initial hospitalization before completion of a full therapeutic course
of antibiotics) is indicated in patients with infective endocarditis (IE) who present with
valve dysfunction resulting in symptoms of heart failure (HF). Reinfection after
prosthetic valve surgery (which occurs in 5-10% of patients, with a significant
percentage of these being injectable drug users) is low relative to the risk of no surgery
in patients with hemodynamic and microbial indications for surgery. Repair rather than
replacement of a valve is always best; however, such repairs are possible in only a
minority of cases, such as when a leaflet perforation occurs without extensive leaflet
destruction or annular involvement. Prompt surgical consultation should be obtained in
all cases of IE to assist with assessment of the need for surgical treatment and to help
judge the timing of surgery.

KKUH
Collected by:
Dr Hani Abdullah
Dr Fatma Hadi
Dr Salem Boresa
Dr Rami Elesali
Dr Naeif Almagal
1/1/2020
8

There is no role for sacubitril/valsartan in HF related to acute valvular disease.


Rifampin may be used in cases of IE for long-term suppressive therapy, but antibiotics
are not a substitute for valve surgery in a patient with HF from IE.
The patient is not in cardiogenic shock and thus intra-aortic balloon pump support is not
indicated; furthermore, even if indicated, it would be bridge to definite valve surgery.
Repeating a transesophagael echocardiogram in 6 weeks is not the favored approach
due to the high mortality of left-sided endocarditis with hemodynamically significant
valvular dysfunction.
Answer B
Key Point
Surgical management of infective endocarditis has the following objectives: completely
excise all infected and necrotic tissue, remove and/or replace all infected prosthetic
material, and reconstruct cardiac structures to restore proper physiology.

Question 6
6-A 29-year-old female is referred by her internist for evaluation of a newly identified
murmur. The patient reports no cardiovascular symptoms and has no significant
medical history except for a known penicillin allergy. Her only prescribed medicine is an
oral contraceptive. She denies recreational drug use.
On physical examination, there is a systolic click followed by a grade 2/6 late systolic
murmur.
Her transthoracic echocardiogram demonstrates myxomatous mitral valve disease
(MVD) with moderate regurgitation and mild left atrial enlargement. The left ventricular
end-systolic dimension is 32 mm and ejection fraction is 65%.

Which of the following is required prior to dental procedures in this patient?


KKUH
Collected by:
Dr Hani Abdullah
Dr Fatma Hadi
Dr Salem Boresa
Dr Rami Elesali
Dr Naeif Almagal
1/1/2020
9

A. No antibiotic prophylaxis.
B. Clindamycin 600 mg.
C. Cephalexin 2 g.
D. Amoxicillin 2 g.
E. Metronidazole 500 mg.

The most current guidelines do not recommend antibiotic prophylaxis for endocarditis
for myxomatous MVD. Endocarditis prophylaxis is only recommended for conditions
associated with the highest risk of adverse outcome from endocarditis. These conditions
include prosthetic cardiac valve or prosthetic valve repair material, prior history of
infective endocarditis, cardiac transplant recipients with valvulopathy, completely
repaired congenital heart disease (CHD) with percutaneous or surgical repair occurring
within the previous 6 months, repaired CHD with residual shunts or defects that impair
endothelialization of prosthetic material, and unrepaired cyanotic CHD.
For patients with these conditions, antibiotic prophylaxis is recommended for dental
procedures that involve manipulation of the gingival tissues, the periapical region of the
teeth, or perforation of oral mucosa. Antibiotic prophylaxis also may be considered for
procedures that involve incision of the respiratory mucosa (e.g., tonsillectomy or
adenoidectomy) and for genitourinary, skin, or gastrointestinal procedures that involve
infected areas. Patients undergoing diagnostic procedures in the absence of these
conditions do not require antibiotic prophylaxis.
Options for antibiotic prophylaxis for endocarditis prior to dental procedures are shown
in Tables 1 and 2. Answer A

KKUH
Collected by:
Dr Hani Abdullah
Dr Fatma Hadi
Dr Salem Boresa
Dr Rami Elesali
Dr Naeif Almagal
1/1/2020
10

KKUH
Collected by:
Dr Hani Abdullah
Dr Fatma Hadi
Dr Salem Boresa
Dr Rami Elesali
Dr Naeif Almagal
1/1/2020
11

Question 7
7-A 44-year-old male comes to your clinic to establish general cardiology care after a
mitral valve replacement with a mechanical prosthesis. His current medications are
warfarin and metoprolol. His blood pressure is 120/80 mm Hg and heart rate is 60 bpm.
His physical exam reveals a soft systolic murmur and crisp metallic valve sounds.

He is inquiring if the direct oral anticoagulation agents advertised on the internet and
television are a reasonable choice for him as he does not wish to be on warfarin.

Which of the following do you recommend?


KKUH
Collected by:
Dr Hani Abdullah
Dr Fatma Hadi
Dr Salem Boresa
Dr Rami Elesali
Dr Naeif Almagal
1/1/2020
12

A. Continue warfarin and add aspirin.


B. Switch warfarin to dabigatran.
C. Continue warfarin and add dipyridamole.
D. Continue warfarin and add clopidogrel.

Effective oral antithrombotic therapy in patients with mechanical heart valves requires
continuous vitamin K antagonist anticoagulation (VKA) with an international normalized
ratio (INR) in the target range. It is preferable to specify a single INR target for each
patient and to recognize that the acceptable range includes 0.5 INR units on each side
of this target. A specific target is preferable because it reduces the likelihood of patients
having INR values consistently near the upper or lower boundary of the range. In
addition, fluctuations in INR are associated with an increased incidence of complications
in patients with prosthetic heart valves, so patients and caregivers should strive to attain
the specific INR value.

The effects of VKA anticoagulation vary with the specific drug, absorption, various
foods, alcohol, other medications, and changes in liver function. Most of the published
studies of VKA therapy used warfarin, although other coumarin agents are used on a
worldwide basis. In clinical practice, a program of patient education and close
surveillance by an experienced healthcare professional, with periodic INR
determinations, is necessary. Patient monitoring through dedicated anticoagulation
clinics results in lower complication rates than those seen with standard care and is cost
effective because of lower rates of bleeding and hemorrhagic complications. Periodic
direct patient contact and telephone encounters with the anticoagulation clinic
pharmacists or nurses are equally effective in reducing complication rates. Self-
monitoring with home INR measurement devices is another option for educated and
motivated patients.

Aspirin is recommended for all patients with prosthetic heart valves, including those with
mechanical prosthetic valves receiving VKA therapy. Even with the use of VKA, the risk
of thromboemboli is 1-2% per year.

The addition of aspirin 100 mg daily to oral VKA-anticoagulation decreases the


incidence of major embolism or death (1.9% vs. 8.5% per year; p < 0.001), with the
stroke rate decreasing to 1.3% per year versus 4.2% per year (p < 0.027), and overall
mortality to 2.8% per year versus 7.4% per year (p < 0.01). The addition of low-dose
aspirin (75 mg to 100 mg per day) to VKA therapy (INR 2.0 to 3.5) also decreases
mortality due to other cardiovascular diseases. The combination of low-dose aspirin and
a VKA is associated with a slightly increased risk of minor bleeding such as epistaxis,
bruising, and hematuria, but the risk of major bleeding does not differ significantly
between those who received aspirin (8.5%) versus those who did not (6.6%; p = 0.43).
The risk of gastrointestinal irritation and hemorrhage with aspirin is dose dependent
KKUH
Collected by:
Dr Hani Abdullah
Dr Fatma Hadi
Dr Salem Boresa
Dr Rami Elesali
Dr Naeif Almagal
1/1/2020
13

over the range of 100 mg to 1,000 mg per day, but the antiplatelet effects are
independent of dose over this range. The addition of aspirin (75 mg to 100 mg per day)
to a VKA should be strongly considered unless there is a contraindication to the use of
aspirin (i.e., bleeding or aspirin intolerance). This combination is particularly appropriate
in patients who have had an embolus while on VKA therapy with a therapeutic INR,
those with known vascular disease, and those who are known to be particularly
hypercoagulable.

The one randomized controlled study of dabigatran compared with warfarin in patients
with mechanical valves was stopped early due to higher rates of bleeding and
thrombosis in the dabigatran arm. No other direct oral anticoagulants have been studied
in this patient population. There is no role for clopidogrel or dipyridamole in the
management of mechanical heart valves. Question

Question 8
8-You are meeting a 54-year-old male who has recently relocated and seeks to
establish cardiovascular care. He underwent aortic valve replacement with a low-profile
bileaflet tilting disk valve and simultaneous aortic root replacement at the age of 51 due
to bicuspid aortic valve disease with an ascending aortic aneurysm. His primary concern
for this visit is to seek an alternative to warfarin, because he was admitted with
gastrointestinal bleeding from gastritis 2 months ago. He has no other medical
problems. His current medications are aspirin 81 mg and warfarin dosed to maintain an
international normalized ratio (INR) of 2.0-3.0. His cardiac exam reveals crisp S1 and
S2 sounds.

Which of the following do you recommend for this patient?


A. Continue warfarin and aspirin.
B. Adjust warfarin for an INR of 1.5-2.5.
C. Stop aspirin.
D. Replace aspirin with clopidogrel.
E. Replace warfarin with dabigatran.

KKUH
Collected by:
Dr Hani Abdullah
Dr Fatma Hadi
Dr Salem Boresa
Dr Rami Elesali
Dr Naeif Almagal
1/1/2020
14

Effective oral antithrombotic therapy in patients with mechanical heart valves requires
continuous vitamin K antagonist (VKA) anticoagulation with an INR in the target range.
It is preferable to specify a single INR target for each patient and to recognize that the
acceptable range includes 0.5 INR units on each side of the target. A specific target is
preferable because it reduces the likelihood of patients having INR values consistently
near the upper or lower boundary of the range. The effects of VKA anticoagulation vary
with the specific drug, absorption, various foods, alcohol, other medications, and
changes in liver function. Most of the published studies of VKA therapy used warfarin,
although other coumarin agents are used on a worldwide basis.

In clinical practice, a program of patient education and close surveillance by an


experienced healthcare professional, with periodic INR determinations, is necessary.
Patient monitoring through dedicated anticoagulation clinics results in lower
complication rates than those seen with standard care and is cost effective because of
lower rates of bleeding and hemorrhagic complications. Periodic direct patient contact
and telephone encounters with the anticoagulation clinic pharmacists or nurses are
equally effective in reducing complication rates. Self-monitoring with home INR
measurement devices is another option for educated and motivated patients.

A randomized controlled trial of dabigatran compared with warfarin for patients with
mechanical prosthetic valves was stopped early due to higher rates of both bleeding
and thrombosis in patients receiving dabigatran. No other direct oral anticoagulants
have been studied in this patient population.

Aspirin and not P2Y12 inhibitors, such as clopidogrel, have been used alongside
warfarin in clinical trials for Food and Drug Administration approval of currently available
mechanical valves and this combination remains recommended for all patients with
mechanical valves in either the aortic or mitral position.

In a patient with a history of gastrointestinal bleeding that has been appropriately


treated, it would not be appropriate to withhold antiplatelet or anticoagulant agents.
Answer A
Key Point
Aspirin alone or direct anticoagulants are not an alternative to warfarin for patients with
mechanical prosthetic valves.

KKUH
Collected by:
Dr Hani Abdullah
Dr Fatma Hadi
Dr Salem Boresa
Dr Rami Elesali
Dr Naeif Almagal
1/1/2020
15

Question 9
9-A 48-year-old male presents for evaluation of bicuspid aortic stenosis because of
worsening shortness of breath over the past few months. He is an avid motorcyclist and
despite a recent accident, would never consider not riding. His echocardiogram 6
months ago showed normal biventricular function, a peak transaortic velocity of 4.2
m/sec, peak transaortic gradient of 72 mm Hg, mean gradient of 48 mm Hg, and
calculated aortic valve area of 0.8 cm2. His aortic root is a normal size. His past medical
history is significant for hypothyroidism. On exam today his blood pressure is 128/66
mm Hg, heart rate is 72 bpm, and he has a normal S1 and a late peaking harsh systolic
murmur. His lungs are clear to auscultation bilaterally and he has no peripheral edema.

Which of the following is the next most appropriate step in the care of this patient?
A. Transesophageal echocardiogram.
B. Repeat transthoracic echocardiogram in 1 year.
C. Bioprosthetic aortic valve replacement.
D. Exercise stress testing.
E. Mechanical aortic valve replacement.

This patient has severe aortic stenosis, has now developed symptoms and should
undergo aortic valve replacement (AVR). The choice of which type of valve,
bioprosthetic versus mechanical, is an individual one and should be based on the
individual's preferences, risks of long-term anticoagulation, and contraindications to
anticoagulation. A bioprosthesis is recommended in patients of any age for whom
anticoagulant therapy is contraindicated, cannot be managed appropriately, or is not
desired (Class 1, Level of Evidence C). Per the 2017 update to the valvular heart
disease guideline, a bioprosthetic valve is reasonable for patients 50-70 years of age
based on individual patient factors and preferences. While the risk for reoperation with
bioprosthetic valves is greater in younger patients, this must be balanced against the
risk of bleeding such as in this patient who is an avid motorcyclist. Transesophageal
echocardiography would not alter the management of this patient. Since this patient is
now symptomatic, AVR is indicated. Exercise stress testing is reasonable to assess
physiological changes with exercise and to confirm the absence of symptoms in
reportedly asymptomatic patients with a calcified aortic valve and an aortic velocity ≥4.0
m/sec or a mean pressure gradient of ≥40 mm Hg (stage C). This patient reports
symptoms, and exercise stress testing is contraindicated in this setting (Class III).
Answer C

Key Point
Selection of valve procedure (repair vs. replacement, surgical vs. transcatheter) and
prosthetic valve type (mechanical vs. bioprosthetic) has to be individualized according
to the patient’s characteristics, including age, surgical risk, and personal preferences
KKUH
Collected by:
Dr Hani Abdullah
Dr Fatma Hadi
Dr Salem Boresa
Dr Rami Elesali
Dr Naeif Almagal
1/1/2020
16

Question 10

10- A 71-year-old male with a history of a bioprosthetic aortic valve replacement 16


years ago for endocarditis returns for routine follow-up. He denies any symptoms of
exertional dyspnea, chest pressure, or dizziness.

His examination reveals a grade 3/6 mid-peaking systolic ejection murmur and a mild
diastolic murmur. An S4 is present.

His echocardiogram shows a left ventricular (LV) ejection fraction of 55%, an LV end-
diastolic diameter of 4.9 cm, an LV end-systolic diameter of 2.3 cm, and LV wall
thickness of 1.1 cm. Peak aortic velocity is 4.3 m/sec and mean aortic gradient is 44
mm Hg. The aortic valve area is 0.9 cm2 with mild aortic regurgitation and there is no
tricuspid regurgitation jet to estimate pulmonary pressures.

Based on these data, which of the following is the next best step in the management of
this patient?
A. Dobutamine stress echocardiogram.
B. Exercise testing.
C. Transesophageal echocardiogram.
D. Refer for aortic valve replacement.

The indications for intervention in prosthetic valve stenosis are the same as those for
native stenosis of the aortic or mitral valve. This patient has severe prosthetic aortic
stenosis (AS) without symptoms. Thus, exercise stress testing is as Class IIa indication
in this setting. The inability to augment systolic blood pressure by 20 mm Hg or the
development of symptoms are adverse prognostic signs that would merit intervention.

Patients with prosthetic valve AS merit referral to a heart valve team for consideration of
percutaneous or surgical intervention.

Neither transesophageal echocardiogram nor dobutamine stress echocardiogram are


indicated when the transthoracic echocardiogram already demonstrates severe AS.

Aortic valve replacement is not indicated in the absence of symptoms or high-risk


findings on stress testing. Answer B
Key Point
Doppler echocardiography (transthoracic echocardiography and transesophageal
echocardiography) is the method of choice for the diagnosis of prosthetic valve
dysfunction (stenosis and regurgitation).
KKUH
Collected by:
Dr Hani Abdullah
Dr Fatma Hadi
Dr Salem Boresa
Dr Rami Elesali
Dr Naeif Almagal
1/1/2020
17

Question 11
11-You are seeing a 40-year-old female with flushing and diarrhea as well as
progressive ankle swelling. She has gained 20 pounds in the last 6 months and says
her clothes are fitting tight around the waist. She has no other medical problems and
takes no medications. Her physical exam reveals a blood pressure of 106/70 mm Hg, a
pulse of 88 bpm, and respirations of 14 per minute. Her jugular veins are distended with
prominent "v" waves. There is a soft, early systolic murmur along her right lower sternal
border. Her liver is pulsatile and palpable two finger-breadths below the costal margin.
There is 3+ bilateral lower extremity edema up to the thighs. Her laboratory evaluations
include a normal thyroid stimulating hormone, normal electrolytes and renal function,
elevated total bilirubin, and elevated 5-HIAA.

Which of the following echocardiographic findings would confirm your clinical diagnosis?
A. Hepatic vein flow reversal.
B. A peak tricuspid regurgitant velocity of 4 m/s.
C. A systolic notch on the pulmonary valve.
D. Pulmonary vein flow reversal.
E. Inferior vena cava collapse with inspiration.

Causes of primary tricuspid regurgitation (TR) include radiation, Ebstein's anomaly,


infective endocarditis, cardiovascular implanted electronic devices, and carcinoid
syndrome, which is what this patient's clinical presentation suggests. Advanced degrees
of TR may be detected on physical examination by the appearance of elevated “c-V”
waves in the jugular venous pulse (JVP), a systolic murmur at the lower sternal border
that increases in intensity with inspiration, and a pulsatile liver edge. In many patients,
characteristic findings in the JVP are the only clues to the presence of advanced TR,
because a murmur may be inaudible even with severe TR. Symptoms include fatigue
from low cardiac output, abdominal fullness, edema, and palpitations, particularly if atrial
fibrillation is also present. Progressive hepatic dysfunction may occur due to the
elevated right atrial pressure, and thus assessment of liver function is useful in patients
with advanced degrees of TR.

Transothoracic echocardiography can distinguish primary from functional TR, define any
associated left-sided valvular and/or myocardial disease, and provide an estimate of
pulmonary artery systolic pressure. Characterization of severity of TR relies on an
integrative assessment of multiple parameters.

Hepatic vein flow reversal is seen in severe TR, which is the correct answer.

Systolic notching of the pulmonic valve is seen in severe pulmonary hypertension and
not expected in cases of primary TR related to carcinoid syndrome.

KKUH
Collected by:
Dr Hani Abdullah
Dr Fatma Hadi
Dr Salem Boresa
Dr Rami Elesali
Dr Naeif Almagal
1/1/2020
18

The inferior vena cava would be plethoric, not collapsed, in severe TR.

Pulmonary vein flow reversal is a sign of severe MR and would not be expected from a
right-sided valvular lesion.

The peak tricuspid regurgitant velocity reflects the right ventricular-right atrial pressure
gradient and does not reflect the severity of TR. Answer A
Key Point
In addition to Doppler color jet area, quantitative measures should be integrated to
determine tricuspid regurgitation (TR) severity, including Doppler characteristics of the
TR jet, jet density, contour by continuous wave, width of the vena contracta, systolic
hepatic vein flow, proximal isovelocity surface area (PISA)–derived regurgitant fraction
and effective regurgitant orifice area (EROA), as well as right atrium and ventricle size
and function. Systolic hepatic vein flow reversal is the strongest correlate for severe TR.

Question 12
12-A 55-year-old female presents with a 6-month history of progressive dyspnea on
exertion. She has a history of hypertension and hyperlipidemia. Her medications include
amlodipine 5 mg daily and atorvastatin 20 mg daily. On examination, her heart rate is 76
bpm and irregular; her blood pressure is 126/70 mm Hg. Her jugular venous pressure is
8 cm H20 with prominent v waves. Her lungs are clear. Her cardiac exam reveals a brief,
high-pitched sound after S2 followed by a low-pitched rumble, best heard at the apex at
held expiration. Her extremities have no edema.

A transthoracic echocardiogram (TTE) reveals an ejection fractrion of 60% with


thickened mitral leaflets, reduced motion during diastole, and doming. The mean mitral
valve area is 1.2 cm2. There is moderate tricuspid regurgitation with an estimated right
ventricular systolic pressure of 55 mm Hg.

Which of the following studies is necessary to determine the correct treatment approach
for this patient?
KKUH
Collected by:
Dr Hani Abdullah
Dr Fatma Hadi
Dr Salem Boresa
Dr Rami Elesali
Dr Naeif Almagal
1/1/2020
19

A. Cardiopulmonary exercise stress test.


B. Exercise stress echocardiogram.
C. Transesophageal echocardiogram.
D. Cardiac magnetic resonance imaging.
E. Right heart catheterization.

This patient has severe symptomatic mitral stenosis (MS) with atrial fibrillation and
pulmonary hypertension. Percutaneous balloon mitral commissurotomy is the treatment
of choice for severe symptomatic MS over surgical commissurotomy as long as the
valve is mobile, relatively thin, and free of calcium and there is no left atrial clot or more
than mild mitral regurgitation (MR).

The 2014 American College of Cardiology/American Heart Association guideline on the


management of valvular heart disease note that a transesophageal echocardiogram
(TEE) should be performed in patients considered for percutaneous mitral balloon
commissurotomy (PMBC) to assess the presence or absence of left atrial thrombus and
to further evaluate the severity of MR. In the vast majority of patients with MS, valve
morphology and lesion severity can be obtained with TTE. A key exception is in patients
being considered for PMBC, in whom left atrial cavity and appendage thrombi must be
excluded. Although a TTE may identify risk factors for thrombus formation, TTE has
poor sensitivity for detecting such thrombi, thus mandating a TEE before PMBC.
Although TTE is generally accurate in grading MR, TEE may offer additional
quantitation and assurance that MR >2+ is not present, which generally precludes
PMBC.

In the contemporary era, adequate assessment of MS and associated lesions can be


obtained in the vast majority of patients by TTE, occasionally supplemented by TEE,
and thus cardiac catheterization is rarely indicated. Even if it were to be performed, right
heart catheterization alone would not be the correct approach, as simultaneous
pressure measurements in the left ventricle (LV) and left atrium, via transseptal
catheterization, are necessary because the LV to pulmonary wedge gradient will
overestimate the true transmitral gradient due to phase delay and delayed transmission
of pressure changes.

Exercise testing with Doppler or invasive hemodynamic assessment is recommended to


evaluate the response of the mean mitral gradient and pulmonary artery pressure in
patients with MS when there is a discrepancy between resting Doppler
echocardiographic findings and clinical symptoms or signs. However, there is no
discrepancy in this case and thus exercise testing is not indicated.

A cardiopulmonary exercise stress test measures maximum oxygen consumption and is


useful in differentiating cardiac from pulmonary causes of dyspnea and offers prognostic
KKUH
Collected by:
Dr Hani Abdullah
Dr Fatma Hadi
Dr Salem Boresa
Dr Rami Elesali
Dr Naeif Almagal
1/1/2020
20

information for end-stage heart failure patients contemplating heart transplantation.


However, this test is not useful in the evaluation or management of patients with MS.

While cardiac magnetic resonance imaging is useful in many cardiac conditions, it plays
little role in the evaluation of patients with MS. Answer C
Key Point
Percutaneous mitral balloon commissurotomy is indicated in symptomatic patients or
asymptomatic patients with pulmonary hypertension, moderate or severe stenosis, and
favorable valve morphology in the absence of left atrial thrombus or moderate to severe
mitral regurgitation.

Question 13
13- A 40-year-old male with a history of bicuspid aortic valve (BAV) presents to your
office with complaints of exertional dyspnea. He recently joined a gym to help him lose
weight. He has started jogging on a treadmill, however he notes shortness of breath
when running at an incline. He denies chest pain and is able to run on level ground
without difficulty. His physical exam includes a height of 70 inches, weight of 225 lbs,
blood pressure of 112/73 mm Hg, and a heart rate of 82 bpm. His carotid pulses are
prominent and lungs are clear. His heart exam is notable for a holodiastolic murmur.
There is no lower extremity edema. A transthoracic echocardiogram is performed. His
left ventricular ejection fraction is 60%. There is a BAV with aortic regurgitation
(pressure half-time [PHT] 280 msec, vena contracta width 0.5 cm, and effective
regurgitant oriface [ERO] 0.26 cm2). His ascending aorta measures 4.8 cm. These
findings are similar to his echocardiogram performed last year.

What is the next best step in the management of this patient?


A. Genetic testing.
B. Echocardiogram in 6 months.
C. Cardiac surgery referral.
D. Hydralazine 10 mg three times a day.
E. Computed tomography scan in 12 months.

KKUH
Collected by:
Dr Hani Abdullah
Dr Fatma Hadi
Dr Salem Boresa
Dr Rami Elesali
Dr Naeif Almagal
1/1/2020
21

This patient has moderate aortic regurgitation (PHT 200-500 msec, vena contracta
width 0.3-0.6 cm, and ERO 0.1-0.29 cm2). His exertional dyspnea is mild, only occurring
when running at an incline and is likely due to deconditioning given that he has only
recently started exercising and is overweight. The most notable finding in his
presentation is the dilated ascending aorta at 4.8 cm. BAVs are frequently associated
with aortic dilation either at the level of the sinuses of Valsalva or, more frequently, in
the ascending aorta. The incidence of aortic dilation is higher in patients with fusion of
the right and noncoronary cusps. Aortic imaging is recommended annually in patients
with a BAV and significant aortic dilation (>4.5 cm), a rapid rate of change in aortic
diameter (increase of >0.5 cm in a year, and in those with a family history of aortic
dissection (Class I recommendation, Level of Evidence C). Aortic imaging can be by
echocardiogram if there is adequate image quality with visualization of the aorta up to 4
cm distal to the valve. Alternatively computed tomography or magnetic resonance
imaging can provide better spatial resolution and is preferred in patients with poor
echocardiographic windows.

This patient has no indication for cardiac surgery at this point. Surgical intervention is
recommended at a dimension of 5.5 cm or at 5.1-5.5 cm in patients with rapid growth or
a family history of aortic dissection. An echocardiogram at 6 months is too soon. Given
his normal blood pressure, there is no benefit of adding hydralazine. At present, there
are no proven drug therapies that have been shown to reduce the rate of progression of
aortic dilation in patients with aortopathy associated with BAV. In patients with
hypertension, control of blood pressure is warranted. A specific genetic cause has not
been identified yet in patients with BAV and aortopathy and therefore genetic testing is
not recommended. Answer E
Key Point
Patients with aortic regurgitation on the basis of a bicuspid aortic valve should be
evaluated and followed for abnormalities of the aorta.

KKUH
Collected by:
Dr Hani Abdullah
Dr Fatma Hadi
Dr Salem Boresa
Dr Rami Elesali
Dr Naeif Almagal
1/1/2020
22

Question 14
14-You are seeing a 58-year-old female in clinic for follow-up of an abnormal computed
tomography (CT) scan of her chest. She had pleuritic chest pain and was diagnosed
with a pulmonary embolus 2 weeks ago. Since starting on anticoagulant therapy she
feels well. She is a life-long nonsmoker. She has two adult children. Her vital signs are a
pulse of 78 bpm, respirations 12 per minute, and blood pressure of 168/92 mm Hg in
the right arm and 162/91 mm Hg in the left arm. Her height is 64 inches (162 cm) and
weight is 178 pounds (80.7 kg). Her carotid pulses are brisk. The apical impulse is not
displaced; S1 and S2 are normally split without any clicks. The remainder of her exam is
unremarkable. A CT angiogram of her chest demonstrated that her aorta was dilated,
measuring 4.2 cm at the mid-ascending aorta, without a visible dissection flap.

Which of the following is the most appropriate next test for this patient?
A. Transthoracic echocardiogram.
B. Repeat CT in one year.
C. Carotid ultrasound.
D. Magnetic resonance angiography.
E. Coronary angiography.

Patients with dilation of the aortic root or ascending thoracic aorta should be evaluated
for bicuspid aortic valve and/or aortic regurgitation with transthoracic echocardiography.

There are no data to support screening for carotid disease in asymptomatic patients.

Magnetic resonance angiography is a reasonable alternative to CT angiography for


surveillance of aortic diameter, but first any associated valve disease must be identified
before arranging surveillance imaging.

Repeat imaging for stability of the aortic size is appropriate, but an assessment of aortic
valve function is needed prior to planning the schedule and best imaging modality for
that surveillance.

Coronary angiography would be appropriate in patients with aneurysms large enough to


warrant elective repair, generally indicated at aortic diameters of >5.5 cm in sporadic
thoracic aneurysms such as this, but not in this patient without risk factors for rupture.
Answer A
Key Point
Aortic regurgitation may be due to abnormalities of the aortic valve and/or aorta.

KKUH
Collected by:
Dr Hani Abdullah
Dr Fatma Hadi
Dr Salem Boresa
Dr Rami Elesali
Dr Naeif Almagal
1/1/2020
23

Question 15
15-A 29-year-old female presented for prenatal counseling. She exercises by running
three miles 5 days per week without limitations. She has a history of a heart murmur
since childhood and a previous echocardiogram suggesting ‘enlargement of the heart'.

Her physical exam reveals normal jugular venous pressure and contour. Precordial
palpation is unremarkable. Auscultation along the left sternal border reveals a high-
pitched sound just after S1 followed by a murmur.

Which of the following maneuvers will best identify the associated valvular abnormality
in this patient?
A. Standing.
B. Inspiration.
C. Hand grip.
D. Valsalva.
E. Squatting.

This patient has a right-sided ejection click and ejection murmur. Ejection clicks are
high-pitched sounds that occur at the moment of maximal opening of the aortic or
pulmonary valves. They are heard just after the first heart sound. The sounds occur in
the presence of a dilated aorta or pulmonary artery or in the presence of a bicuspid or
flexible stenotic aortic or pulmonary valve. In this case, the ejection click and murmur
are heard only at the left sternal border, consistent with pulmonic rather than aortic
stenosis.

The most helpful distinguishing feature of a pulmonary ejection sound is its decreased
intensity, or even its disappearance during the inspiratory phase of respiration. During
expiration, the valve opens rapidly from its fully closed position; sudden "halting" of this
rapid opening movement is associated with a maximal intensity of the ejection sound.
With inspiration, the increased venous return to the right ventricle (RV) augments the
effect of right atrial systole and causes partial opening of the pulmonary valve prior to
ventricular systole. The lack of a sharp opening movement of the pulmonary valve
explains the decreased intensity of the pulmonary ejection sound during inspiration.
Thus, the best way to confirm that the patient has pulmonic stenosis is to assess the
intensity of the murmur on inspiration.

The other maneuvers listed would not be expected to affect the auscultation of a right-
sided murmur.

Sustained hand grip for 20-30 seconds leads to an increase in systemic vascular
resistance, arterial pressure, cardiac output, and left ventricular (LV) volume and filling
pressure. Hand grip is most useful in differentiating between the ejection systolic
KKUH
Collected by:
Dr Hani Abdullah
Dr Fatma Hadi
Dr Salem Boresa
Dr Rami Elesali
Dr Naeif Almagal
1/1/2020
24

murmur of aortic stenosis and the regurgitant murmur of mitral regurgitation (MR).
Intensity of the murmur of aortic stenosis tends to decrease along with a decreased
transvalvular pressure gradient, while the severity and murmur of MR increase.

During the straining phase, phase 2, of Valsalva there is a decrease in venous return,
RV and LV volumes, stroke volumes, mean arterial pressure, and pulse pressure; this is
associated with a reflex increase in heart rate. The murmur of hypertrophic
cardiomyopathy (HCM) increases in intensity as the LV outflow size decreases with a
decreased venous return. In mitral valve prolapse (MVP) there is an early onset of the
click and murmur due to the decrease in LV volume.

Abrupt standing from the supine position decreases venous return to the heart and,
consequently, RV and LV diastolic volumes and stroke volumes decline. There also
may be a fall in arterial pressure and a reflex increase in heart rate. This will cause
similar effects to phase 2 of a Valsalva maneuver.

Squatting from a standing position is associated with a simultaneous increase in venous


return (preload) and systemic vascular resistance (afterload) and a rise in arterial
pressure. In HCM, intensity of the ejection systolic murmur declines because of an
increased LV volume and arterial pressure, which increase the effective orifice size of
the outflow tract. In patients with MVP there is a delay in the onset of the click and a
shortening of the late systolic murmur. These changes reflect the delay in prolapse
induced by the increase in preload. However, as MR becomes more severe, the
murmur may increase in intensity with squatting because of the increase in afterload.
Answer B
Key Point
The only right-sided auscultatory event that diminishes with inspiration is the pulmonary
ejection click associated with pulmonary valve stenosis.

KKUH
Collected by:
Dr Hani Abdullah
Dr Fatma Hadi
Dr Salem Boresa
Dr Rami Elesali
Dr Naeif Almagal
1/1/2020
25

Question 16
16-A 76-year-old male with a past history of coronary artery bypass grafting 20 years
ago is admitted with progressive dyspnea on exertion. He has diabetes mellitus, stage 3
chronic kidney disease, hypertension, dyslipidemia, and had a stroke 5 years ago that
left him with residual left-sided weakness. His echocardiogram shows a mildly dilated
left ventricle, a left ventricular ejection fraction of 25-30% with regional variations, and
severe aortic stenosis (AS) with a mean gradient of 50 mm Hg and a calculated valve
area of 0.8 cm2. Coronary angiography demonstrates patent grafts and severe native
vessel disease.

What is the most appropriate next step in the management of this patient?
A. Transcatheter aortic valve replacement.
B. Valved apical-aortic conduit.
C. Balloon aortic valvuloplasty.
D. Left ventricular assist device.
E. Dobutamine infusion.

This patient has severe symptomatic AS. The recommendation for either surgical aortic
valve replacement (AVR) or transcather AVR among high-risk patients with severe,
symptomatic AS (stage D), after consideration by a heart valve team, was changed from
Class IIa (Level of Evidence [LOE] B) to Class I (LOE A) in the 2017 guideline for
patients with valvular heart disease.

Percutaneous aortic balloon dilation has an important role in treating children,


adolescents, and young adults with AS, but its role in treating older patients is very
limited. The mechanism by which balloon dilation modestly reduces the severity of
stenosis in older patients is by fracturing calcific deposits within the valve leaflets and, to
a minor degree, stretching the annulus and separating the calcified or fused
commissures. Immediate hemodynamic results include a moderate reduction in the
transvalvular pressure gradient, but the postdilation valve area rarely exceeds 1.0 cm 2.
Despite the modest change in valve area, an early symptomatic improvement usually
occurs. However, serious acute complications, including acute severe aortic
regurgitation, restenosis, and clinical deterioration, occur within 6-12 months in most
patients. Therefore, in patients with AS, percutaneous aortic balloon dilation is not a
substitute for AVR.

Apico-aortic conduits to bypass the diseased aortic valve are a historical curiosity and
no longer performed in the era of transcatheter aortic valve therapy.

Left ventricular assist devices (LVAD) are indicated for end-stage heart failure patients
only when all other therapeutic options have been exhausted. In this case AVR would
be expected to result in improvement in symptoms, myocardial performance, and
KKUH
Collected by:
Dr Hani Abdullah
Dr Fatma Hadi
Dr Salem Boresa
Dr Rami Elesali
Dr Naeif Almagal
1/1/2020
26

mortality, so AVR would be preferred to LVAD.

Palliative care with inotropic support would not be expected to improve quality of life or
mortality in a patient with severe symptomatic AS. Answer A
Key Point
The primary indication for aortic valve replacement (AVR) is symptoms due to severe
aortic stenosis (AS). AVR is also recommended in asymptomatic patients with severe
AS and a reduction in ejection fraction (<50%), and may be considered with very severe
AS, rapid AS progression, or at the time of other cardiac surgery.

Question 17
17-A 52-year-old male with a history of Marfan syndrome and remote Type A aortic
dissection status post mechanical aortic valve replacement and root repair presents to
the emergency department with 1 week of progressive dyspnea and orthopnea. On
physical exam, he is afebrile, his heart rate is 92 bpm, blood pressure is 148/92 mm Hg,
respiratory rate is 22 breaths per minute, and pulse oximetry is 90% on room air. His
jugular venous pressure is elevated. His lung exam is notable for rales and scattered
wheezing. His heart sounds are distant. There is 1+ bilateral lower extremity edema. His
electrocardiogram (ECG) shows sinus rhythm with 0.5 mm lateral ST depressions. His
laboratory results reveal a creatinine of 1.2 mg/dl, hemoglobin of 12 g/dl, international
normalized ratio (INR) of 2.2, brain natriuretic peptide of 1200 pg/ml (normal <100), and
troponin I of 0.4 ng/ml (normal <0.04). His transthoracic echocardiogram (TTE) shows a
grossly normal left ventricular ejection fraction with poor Doppler windows. His chest X-
ray shows bilateral patchy infiltrates.

Which of the following is the next best step in the management of this patient?
A. Two sets of blood cultures.
B. Right heart catheterization with thermodilution.
C. Ventilation-perfusion scan.
D. Transesophageal echocardiogram.
E. Coronary angiogram.

KKUH
Collected by:
Dr Hani Abdullah
Dr Fatma Hadi
Dr Salem Boresa
Dr Rami Elesali
Dr Naeif Almagal
1/1/2020
27

The patient presents with new onset left-sided heart failure (HF) with pulmonary edema.
Given his history of mechanical aortic valve replacement, it is essential to evaluate for
valve thrombosis (even in the presence of a therapeutic INR at present as he may have
been subtherapeutic in the past month). Echocardiography holds a Class I indication for
the evaluation of suspected prosthetic valve dysfunction. TTE allows for the evaluation
of the valve hemodynamics and the detection of valve stenosis or regurgitation. In this
case, the TTE was unable to adequately assess the aortic prosthesis. Leaflet motion
and thrombus may be visualized by TTE in some patients, but transesophageal
echocardiography is more sensitive for the detection of valve dysfunction and
thrombosis. By the data provided, the patient has elevated filling pressures and a right
heart catheterization would not be the best initial step. A gated chest computed
tomography scan or fluroscopy may also be useful to evaluate mechanical prosthesis
motion (Class IIa). A ventilation-perfusion scan is useful in the evaluation of pulmonary
embolism not acute HF. Blood cultures are indicated for suspected endocarditis or
unexplained fever in the setting of a prosthetic valve; however, there is a low suspicion
for infection, and valve thrombus should be evaluated first. The elevated troponin and
mild ST abnormality by ECG is most likely due to demand ischemia secondary to acute
HF. He has no chest pain, therefore coronary angiography should not be the initial
study. Answer D
Key Point
Doppler echocardiography (transthoracic echocardiography and transesophageal
echocardiography) is the method of choice for the diagnosis of prosthetic valve
dysfunction (stenosis and regurgitation).

Question 18
18- A 56-year-old female presents for evaluation of a 2-year history of progressive
dyspnea on exertion, which has worsened over the past 4 months. Presently she cannot
climb a flight of stairs without dyspnea. Her medical history is significant for
hypertension. Her medications include metoprolol succinate 50 mg daily.

On examination her heart rate is 80 bpm and her blood pressure is 136/80 mm Hg. Her
jugular venous pressure is elevated to 9 cm H20 with prominent V waves. A left
KKUH
Collected by:
Dr Hani Abdullah
Dr Fatma Hadi
Dr Salem Boresa
Dr Rami Elesali
Dr Naeif Almagal
1/1/2020
28

parasternal lift is present. The first heart sound is normal in intensity. The pulmonic
component of the second heart sound is increased. Both a soft grade 2/4 diastolic
rumble and a 3/6 holosystolic murmur are heard at the apex. At the left sternal border a
grade 2/6 systolic ejection murmur is present and increases with inspiration. A pulsatile
liver is evident with mild lower extremity edema.

A transthoracic echocardiogram demonstrates an ejection fraction of 65% and fusion of


the mitral valve (MV) commissures. The mean transmitral gradient is 14 mm Hg at a
heart rate of 72 bpm. The calculated valve area by the pressure half-time equation is
1.0 cm2. The MV is pliable with no commissural calcification. Moderate to severe mitral
regurgitation and severe tricuspid regurgitation are present. The right ventricle is mildly
dilated with generalized hypokinesis. The pulmonary artery systolic pressure is
estimated at 60 mm Hg.

Which of the following is the next best step in the management of this patient?
A. MV replacement.
B. Metoprolol succinate 100 mg daily.
C. Exercise stress echocardiogram.
D. Balloon mitral valvuloplasty.

The patient has severe symptomatic mitral stenosis (MS) and regurgitation. Although
her MV is pliable, the presence of moderate or worse mitral regurgitation is a
contraindication to balloon valvuloplasty (Figure 1).

Surgical MV replacement is a Class I indication in patients with severe symptomatic MS


(New York Heart Association Class III-IV) who are not a high risk for surgery and who
are not candidates for mitral valvuloplasty.

Increase in beta-blocker therapy is not adequate for management of severe


symptomatic MS.

An exercise stress echocardiogram would be indicated only if there was a discrepancy


between the clinical findings and the results of the echocardiogram. Answer A

KKUH
Collected by:
Dr Hani Abdullah
Dr Fatma Hadi
Dr Salem Boresa
Dr Rami Elesali
Dr Naeif Almagal
1/1/2020
29

Key Point
Percutaneous mitral balloon commissurotomy is indicated in symptomatic patients or
asymptomatic patients with pulmonary hypertension, moderate or severe stenosis, and
favorable valve morphology in the absence of left atrial thrombus or moderate to severe
mitral regurgitation.

KKUH
Collected by:
Dr Hani Abdullah
Dr Fatma Hadi
Dr Salem Boresa
Dr Rami Elesali
Dr Naeif Almagal
1/1/2020
30

Question 19
19- A 34-year-old male is admitted with a fever. He has felt poorly for several months.
He has had a 10-lb weight loss and recurring night sweats. In the past few weeks, he
has experienced progressive dyspnea. He is on no medications at this time. His social
history includes ongoing intravenous drug use.

His temperature is 38.5° Celcius, heart rate is 103 bpm, blood pressure is 95/50 mm
Hg, and resting oxygen saturation on room air is 96%. His lungs reveal bibasilar fine
rales. His jugular venous pressure is not elevated. His apex is hyperdynamic, and there
is a grade 2/6 systolic murmur and grade 2/4 diastolic murmur along the left sternal
border. The first heart sound (S1) is soft and there is an S3. He has no edema. There
are splinter hemorrhages under the nailbeds on his left fingers.

His echocardiogram reveals a hyperdynamic left ventricle with evidence for severe
aortic regurgitation (AR) and vegetations on his aortic valve (Figure 1).

In addition to broad spectrum antibiotics, which of the following is the most appropriate
next step in the management of this patient?

KKUH
Collected by:
Dr Hani Abdullah
Dr Fatma Hadi
Dr Salem Boresa
Dr Rami Elesali
Dr Naeif Almagal
1/1/2020
31

(Figure 1)
A. Aortic valve replacement.
B. Intravenous esmolol 0.1 mg/kg/min.
C. Intra-aortic balloon pump.
D. Phenylephrine 40 mcg/min.

This patient has evidence of infective endocarditis (IE) of the aortic valve resulting in
severe AR and heart failure (HF). His exam is consistent with AR further supported by
premature closure of the mitral valve (MV) on the m-mode echocardiogram shown.
Early surgical intervention is a Class I indication for patients with IE and valve
dysfunction causing HF.

His echocardiography reveals preclosure of the MV due to the rapidly rising left
ventricular diastolic pressure as a result of his severe AR. This also results in the soft
S1 observed. As opposed to chronic AR, the difference between the aortic and left
KKUH
Collected by:
Dr Hani Abdullah
Dr Fatma Hadi
Dr Salem Boresa
Dr Rami Elesali
Dr Naeif Almagal
1/1/2020
32

ventricular pressures in diastole may be small in acute AR and there may be little
diastolic murmur. Likewise the pulse pressure may not be wide, and there may be none
of the classic hemodynamic findings of chronic AR. Schematic hemodynamic tracings of
chronic versus acute AR are shown in Figure 2.

A transesophageal echocardiogram would be indicated in patients with aortic valve


endocarditis to better assess the vegetation sizes and to help define the presence of an
aortic abscess, but the procedure should not delay surgical intervention and can be
performed intraoperatively.

Severe AR is a contraindication to the intra-aortic balloon pump. His tachycardia is


appropriate for his serious hemodynamic state and beta-blockers therefore would not be
appropriate. In addition, the reduced heart rate from a beta-blocker would increase
diastolic time and the duration of AR per beat. Phenylephrine would increase afterload,
which is contraindicated in severe AR. Answer A

KKUH
Collected by:
Dr Hani Abdullah
Dr Fatma Hadi
Dr Salem Boresa
Dr Rami Elesali
Dr Naeif Almagal
1/1/2020
33

KKUH
Collected by:
Dr Hani Abdullah
Dr Fatma Hadi
Dr Salem Boresa
Dr Rami Elesali
Dr Naeif Almagal
1/1/2020
34

Key Point
Transesophageal echocardiography is indicated for diagnosis and evaluation of
vegetation size, abscess formation, fistula formation, leaflet perforation, or prosthetic
valve dehiscence.

KKUH
Collected by:
Dr Hani Abdullah
Dr Fatma Hadi
Dr Salem Boresa
Dr Rami Elesali
Dr Naeif Almagal
1/1/2020
35

Question 20
20-A 75-year-old male presents with 3 days of chest pain and worsening dyspnea on
exertion. His past medical history includes hypertension and osteoarthritis. His
medications include aspirin 81 mg daily, chlorthalidone 25 mg daily, and amlodipine 5
mg daily.

On examination his blood pressure is 125/60 mm Hg, heart rate is 95 bpm, and jugular
venous pressure is 12 cm. His lungs have bibasilar crackles. There is a harsh systolic
murmur heard throughout the precordium without inspiratory augmentation. At the apex,
there is a musical murmur that is louder after a premature ventricular contraction (PVC).
There is no peripheral edema.

His electrocardiogram shows Q waves in the inferior leads.

Which of the following is most likely to be seen on cardiac catheterization of this


patient?
A. Large right ventricular outflow tract to pulmonary artery pressure gradient.
B. Large V wave on right atrial pressure tracing.
C. Large V wave on pulmonary capillary wedge pressure tracing.
D. Increased pulmonary artery oxygen saturation.
E. Large left ventricle to aorta gradient.

The patient has degenerative or age-related aortic stenosis (AS), the most common
etiology of AS. On physical examination, he demonstrates Gallavardin phenomenon—a
harsh murmur at the base with a musical murmur at the apex. This is due to the high-
frequency components of the AS murmur radiating to the left ventricular (LV) apex. It
can be confused with the murmur of mitral regurgitation (MR) except that the murmur of
AS is a systolic ejection murmur rather than holosystolic. The murmur of AS also
increases with bradycardia or after a pause, such as after a PVC; an MR murmur would
not change.

This is not tricuspid regurgitation, which would be holosystolic and best heard at the left
lower sternal border. Nor is this pulmonic stenosis, which would typically best be heard
at the left upper sternal border. Both of these right-sided murmurs would become louder
with inspiration.

If cardiac catheterization were performed, it would demonstrate a large LV to aorta


pressure gradient, consistent with AS. A large V wave on pulmonary capillary wedge
pressure is consistent with significant MR. A large V wave on the right atrium is
consistent with significant tricuspid regurgitation. Increased pulmonary artery (PA)
oxygen saturation suggests the presence of a left-to-right shunt, such as a ventricular
septal defect (VSD). A VSD murmur is typically continuous and heard at the sternal

KKUH
Collected by:
Dr Hani Abdullah
Dr Fatma Hadi
Dr Salem Boresa
Dr Rami Elesali
Dr Naeif Almagal
1/1/2020
36

border, not the apex. A large right ventricular outflow tract to PA pressure gradient is
consistent with pulmonic stenosis. Answer E
Key Point
As the murmur of aortic stenosis (AS) worsens, the ejection sound and the intensity of
A2 diminish; the murmur peaks later in systole. The AS murmur increases after a
premature ventricular contraction.

Question 21
21-A 43-year-old female with a history of endocarditis status post mechanical aortic
valve replacement (AVR) 3 years prior presents with a 2-week history of progressive
dyspnea on exertion. She has a history of nonadherence to medications.

On examination, she is anxious and short of breath. Her temperature is 36.8 degrees
Celcius, blood pressure is 96/60 mm Hg, and heart rate is 106 bpm. Her oxygen
saturation is 93% (2L nasal cannula). Her lungs have crackles halfway up the lung
fields. There is a soft S1 and S2. There is a 2/6 systolic ejection murmur and a 3/4
diastolic murmur at the left sternal border with an S3 gallop. There is no peripheral
edema.

Her interantional normalized ratio is 1.4.

Her transesophageal echocardiogram (TEE) reveals normal left ventricular (LV) size
with preserved LV function, LV ejection fraction of 60%. The peak velocity is 3.8 m/sec
with a mean aortic gradient of 32 mm Hg. There is severe aortic regurgitation by color
Doppler with a steep pressure half-time slope. Cinefluoroscopy is shown in Video 1.

Intravenous (IV) heparin is started and furosemide 40 mg IV is given.

Which of the following is the next best step in the management of this patient?

Vedio consistent with stuck valve

A. Transcatheter AVR.
KKUH
Collected by:
Dr Hani Abdullah
Dr Fatma Hadi
Dr Salem Boresa
Dr Rami Elesali
Dr Naeif Almagal
1/1/2020
37

B. Tissue plasminogen activator 10 mg IV bolus.


C. Surgical AVR.
D. Vancomycin 1 g IV.

This patient has prosthetic valve thrombosis with heart failure symptoms, a Class I
indication for emergent surgery. Cineflouroscopy or computed tomography is often used
as an adjunct to TEE for the diagnosis of prosthetic valve dysfunction.

Fibrinolytic therapy is a Class IIa indication if there is a right-sided thrombus or for a


small left-sided prosthetic valve thrombus in the setting of New York Heart Association
class I-II symptoms of <14 days onset.

There is no indication of infective endocarditis so empiric vancomycin is not appropriate.

Transcatheter AVR is not indicated for prosthetic valve thrombosis. Answer C


Key Point
Doppler echocardiography (transthoracic echocardiography and transesophageal
echocardiography) is the method of choice for the diagnosis of prosthetic valve
dysfunction (stenosis and regurgitation).

Question 22
22-A 34-year-old female is referred to you with shortness of breath and the new onset of
atrial fibrillation (AF). She has no past medical history and takes no medications. Her
vital signs are a pulse of 100 bpm that is irregular, a blood pressure of 110/66 mm Hg,
and respirations of 14 breaths per minute. Her physical exam reveals a 2/4 blowing
diastolic murmur best heard at the apex in end expiration, with an opening snap in
diastole, and a 3/6 holosystolic murmur heard at the apex and radiating to the axilla.

Transthoracic echocardiography demonstrates normal left ventricular (LV) size with a


LV ejection fraction of 66%. There is mitral stenosis (MS; mean gradient of 15 mm Hg at
98 bpm and a mitral valve area of 1.3 cm2) and mild mitral regurgitation (MR). Her

KKUH
Collected by:
Dr Hani Abdullah
Dr Fatma Hadi
Dr Salem Boresa
Dr Rami Elesali
Dr Naeif Almagal
1/1/2020
38

estimated pulmonary systolic pressures are elevated at 64 mm Hg and her left atrial
size is 49 ml/m2.

Which of the following is the next most appropriate study for this patient?
A. Repeat transthoracic echocardiogram in 6 months.
B. Exercise stress echocardiogram.
C. Transesophageal echocardiography.
D. Right heart catheterization.

Transesophageal echocardiography should be performed in patients planning


percutaneous balloon mitral commissurotomy (PBMC) to assess for the presence and
degree of MR and to rule out left atrial or left atrial appendage thrombus prior to the
procedure (Class I). PBMC is indicated in symptomatic patients with severe MS (Class
I) and in asymptomatic patients with new onset AF and favorable valve morphology
(Class IIB) (Figure 1).

The guideline says that exercise testing or hemodynamic assessment is indicated when
there is a discrepancy between resting echocardiogram and clinical symptoms and
signs in the evaluation of MS. There is no discrepancy regarding the severity of MS in
this case and so no further testing is required prior to planning therapy.

Periodic monitoring would only be appropriate for a patient with asymptomatic severe
MS. Answer C

KKUH
Collected by:
Dr Hani Abdullah
Dr Fatma Hadi
Dr Salem Boresa
Dr Rami Elesali
Dr Naeif Almagal
1/1/2020
39

Key Point
Percutaneous mitral balloon commissurotomy is indicated in symptomatic patients or
asymptomatic patients with pulmonary hypertension, moderate or severe stenosis, and
favorable valve morphology in the absence of left atrial thrombus or moderate to severe
mitral regurgitation.

KKUH
Collected by:
Dr Hani Abdullah
Dr Fatma Hadi
Dr Salem Boresa
Dr Rami Elesali
Dr Naeif Almagal
1/1/2020
40

Question 23
23-A 52-year-old female presents to the emergency department with acute shortness of
breath.

Her vital signs include a heart rate of 110 bpm, a blood pressure of 90/60 mm Hg, and a
room air saturation of 90%. She is tachypneic sitting upright. There are rales bilaterally
and a grade 2/6 holosystolic murmur with an associated S3.

Her echocardiogram is shown (Videos 1 and 2).

She undergoes cardiac catheterization with the following pertinent findings:

• Normal coronaries
• Right atrial pressure mean of 10 mm Hg
• Pulmonary artery pressure 65/35 mm Hg (mean 45 mm Hg)
• Pulmonary capillary wedge pressure V wave of 40 mm Hg (mean of 33 mm Hg)
• Aortic gradient 100/80 mm Hg
• Left ventricular pressure 100/29 mm Hg
• Cardiac output 3.2 L/min
• Cardiac index 1.8 L/min/m2

Which of the following is the next best step in the management of this patient?
Video 1 : TTE Apical 4ch , flail post leaflet
Video 2: With colour consistent with severe MR anteriorly directed
A. Norepinephrine 5 mcg/kg/min.
B. Inhaled nitric oxygen 10 ppm.
C. Percutaneous mitral valve repair.
D. Surgical mitral valve repair/replacement.
E. Percutaneous left ventricular assist device.

This woman has acute severe mitral regurgitation from chordal rupture. The correct
answer is urgent surgical intervention.

Medical management may include diuresis and afterload reduction with vasodilators or
an intra-aortic balloon pump, but should be done while preparing for urgent surgical
intervention. Norepinephrine is not indicated because it offers inotropic support with
vasoconstriction. Neither percutaneous left ventricular assist device nor percutaneous
mitral valve repair have been studied in this setting.

Her pulmonary hypertension is secondary to her high pulmonary capillary wedge


KKUH
Collected by:
Dr Hani Abdullah
Dr Fatma Hadi
Dr Salem Boresa
Dr Rami Elesali
Dr Naeif Almagal
1/1/2020
41

pressure (group II pulmonary hypertension) so there is no indication for pulmonary


vasodilator therapy. Answer D

Question 24
24-The patient is an 85-year-old male who presents to the emergency department with
dyspnea and malaise. He has a past medical history of severe aortic stenosis status
post transcatheter aortic valve replacement approximately 5 years ago. He describes
increasing fatigue, dyspnea, and a low grade fever for the past 7 days. On exam, his
temperature is 38.1 degrees Celsius (100.6 degrees Fahrenheit), heart rate is 98 bpm,
blood pressure is 112/50 mm Hg, respiratory rate is 18 respirations per minute, pulse
oximetry is 96% on room air. He has normal S1 and S2 heart sounds with a 2/4
decrescendo diastolic murmur present along the left sternal border. His lungs are clear.
The remainder of his physical examination is normal.

In addition to a transthoracic echocardiogram, which of the following is the most


appropriate next step in the management of this patient?
A. Empiric antibiotics.
B. Obtain two sets of blood cultures.
C. Administer intravenous furosemide.
D. An influenza nasal swab.
E. A chest X-ray.

The patient has a prosthetic aortic valve and is at risk for infective endocarditis (IE). It is
important that two sets of blood cultures are obtained in patients who are at risk for IE,
including those with congenital or acquired valvular heart disease, previous IE,
prosthetic heart valves, certain congenital or heritable heart malformations,
immunodeficiency states, or who are injection drug users. Blood cultures are positive in
>90% of patients with IE and should be obtained at separate time intervals prior to the
administration of antibiotics. This patient has an unexplained fever for >48 hours and a
left-sided regurgitant murmur with a high likelihood for IE. Thus, the most appropriate
next step in management is to obtain two sets of blood cultures.

Blood cultures must be obtained before starting empiric antibiotics. He has no evidence
KKUH
Collected by:
Dr Hani Abdullah
Dr Fatma Hadi
Dr Salem Boresa
Dr Rami Elesali
Dr Naeif Almagal
1/1/2020
42

of pulmonary edema and therefore furosemide is not warranted. While a chest X-ray
and a flu swab may be considered, neither are the essential first steps in management
of this patient. Answer B
Key Point
The diagnosis of infective endocarditis is based on a constellation of history, clinical
findings, laboratory studies (particularly blood cultures), and echocardiography. Use of
an imaging algorithm reduces unnecessary use of echocardiography.

Question 25
25-A 36-year-old female presents to you for evaluation. She has a known history of
mitral valve prolapse (MVP). An echocardiogram one month prior shows an ejection
fraction of 70% with bileaflet MVP and moderate mitral regurgitation (MR). She is able
to hike one hour on the weekends without limitations.

On examination, her body mass index is 30 kg/m2, heart rate is 70 bpm, blood pressure
is 128/70 mm Hg, jugular venous pressure is 2 cm H 20, and her lungs are clear. Her
cardiac exam shows a mid-systolic click and late-systolic murmur radiating to the axilla.
Her extremities are warm without edema.

Which of the following do you recommend for this patient?


A. Losartan 25 mg daily.
B. Lisinopril 5 mg daily.
C. Nifedipine sustained release 30 mg daily.
D. Sacubitril/valsartan 24/26 mg twice daily.
E. No change in therapy.

Vasodilator therapy is not indicated for normotensive asymptomatic patients with


chronic primary MR and normal left ventricular (LV) systolic function.

Because vasodilator therapy appears to be effective in acute severe symptomatic MR, it


KKUH
Collected by:
Dr Hani Abdullah
Dr Fatma Hadi
Dr Salem Boresa
Dr Rami Elesali
Dr Naeif Almagal
1/1/2020
43

seems reasonable to attempt afterload reduction in chronic asymptomatic MR with


normal LV function in an effort to forestall the need for surgery. However, the results
from the limited number of trials addressing this therapy have been disappointing,
demonstrating little or no clinically important benefit. Conversely, because vasodilators
decrease LV size and mitral closing force, they may increase MVP, worsening rather
than decreasing the severity of MR. The foregoing does not apply to patients with
concomitant hypertension. Hypertension must be treated because of the well known
morbidity and mortality associated with that condition and because increased LV
systolic pressure by itself increases the systolic transmitral gradient and worsens the
severity of MR.

The correct answer is no therapy as she is not hypertensive so vasodilator therapy with
an angiotensin-converting enzyme inhibitor, angiotensin-receptor blocker, angiotensin
receptor–neprilysin inhibitor, or nifedipine is not indicated.
Answer E
Key Point
Echocardiography is the most useful diagnostic test to inform the etiology and severity
of mitral regurgitation. Echocardiographic grading of more-than-trivial mitral
regurgitation should include quantitative criteria beyond color jet dimensions, particularly
if jets are nonholosystolic or eccentric.

Question 26
26-A 22-year-old female presents for routine follow-up of mitral stenosis (MS). She was
hospitalized with a prolonged febrile illness at the age of 13. She currently runs 4 miles
in 35 minutes several times per week. She denies palpitations. She is on no medication
except birth control pills.

On physical examination she is 67 inches tall and weighs 127 lbs. Her blood pressure is
122/72 mm Hg and her heart rate is regular at 76 bpm. Her lungs are clear. Her jugular
venous pressure is 4 cm H2O. The first heart sound is increased. Both components of
the second heart sound are normal. A soft early diastolic rumble is present that is
preceded by a crisp opening snap. No peripheral edema is present.
KKUH
Collected by:
Dr Hani Abdullah
Dr Fatma Hadi
Dr Salem Boresa
Dr Rami Elesali
Dr Naeif Almagal
1/1/2020
44

Her echocardiogram shows fusion of the mitral commissures. The mean transmitral
gradient is 4 mm Hg at a heart rate of 88 bpm. The calculated mitral valve (MV) area is
2.3 cm2 by the pressure half-time method. Trivial mitral regurgitation is present. The MV
appears pliable (echocardiographic score = 4).

Which of the following is the next best step in the care of this patient?
A. Vitamin K antagonist therapy.
B. Percutaneous balloon mitral valvotomy.
C. Oral penicillin V 250 mg twice daily.
D. Surgical MV replacement.
E. Transesophageal echocardiography.

The correct answer is penicillin prophylaxis which should be initiated the time of
presentation.

Figure 1 outlines the guideline options that include oral penicillin V twice daily, monthly
benzathine penicillin G intramuscular injection, or daily sulfadiazine. Figure 2 outlines
the duration of therapy. For those patients with residual valvular disease, the
recommended duration is to continue penicillin prophylaxis for 10 years from the last
episode of acute rheumatic fever or until the age of 40. If there was acute carditis, but
no residual valve disease, the recommendation is for 10 years or until age 21
(whichever is longer). If there was rheumatic fever without carditis, the recommendation
is for 5 years or until the age of 21 (whichever is longer).

The patient has evidence of rheumatic MS on physical examination and


echocardiography. However, the MS is mild and she is asymptomatic with no evidence
of atrial fibrillation or pulmonary hypertension. Thus, there is no indication for either
percutaneous or surgical intervention.

A transesophageal echocardiogram would be indicated to rule out left atrial thrombus in


patients being considered for percutaneous balloon mitral valvotomy and is thus not
indicated in this case.

A vitamin K antagonist would be required if atrial arrhythmias were present or there was
evidence for emboli or an atrial thrombus. Answer C

KKUH
Collected by:
Dr Hani Abdullah
Dr Fatma Hadi
Dr Salem Boresa
Dr Rami Elesali
Dr Naeif Almagal
1/1/2020
45

KKUH
Collected by:
Dr Hani Abdullah
Dr Fatma Hadi
Dr Salem Boresa
Dr Rami Elesali
Dr Naeif Almagal
1/1/2020
46

Question 27
27-A 52-year-old male with a history of hypertension and remote occult gastrointestinal
bleeding presumed to be secondary to atrioventricular malformation is referred to your
clinic for evaluation of a new murmur. His physical examination is notable for a blood
pressure of 126/74 mm Hg, heart rate of 72 bpm, and a late peaking systolic murmur.
His echocardiogram reveals a bicuspid aortic valve (BAV) with moderate to severe
aortic stenosis and a dilated ascending aorta. He is referred for surgical evaluation.
KKUH
Collected by:
Dr Hani Abdullah
Dr Fatma Hadi
Dr Salem Boresa
Dr Rami Elesali
Dr Naeif Almagal
1/1/2020
47

What is the next best step in the management of this patient?


A. BAV replacement.
B. Transcatheter aortic valve replacement.
C. Mechanical aortic valve replacement.
D. Ascending aorta.
E. Medical management.

In patients who are being treated with long-term vitamin K antagonist (VKA)
anticoagulation before valve surgery, a mechanical valve may be appropriate, given its
greater durability compared with a bioprosthetic valve and the need for continued VKA
anticoagulation even if a bioprosthetic valve is implanted. However, if interruption of
VKA therapy is necessary for noncardiac procedures, bridging therapy with other
anticoagulants may be needed if a mechanical valve is present, whereas stopping and
restarting VKA therapy for other indications may be simpler. Specific clinical
circumstances, comorbid conditions, and patient preferences should be considered
when deciding between a bioprosthetic and mechanical valve in patients receiving VKA
therapy for indications other than the prosthetic valve itself. Answer A
Key Point
Selection of valve procedure (repair vs. replacement, surgical vs. transcatheter) and
prosthetic valve type (mechanical vs. bioprosthetic) has to be individualized according
to the patient’s characteristics, including age, surgical risk, and personal preferences.

Question 28
28-A 55-year-old female is referred to you for evaluation and treatment of aortic
regurgitation (AR). She has a known history of bicuspid aortic valve (BAV). She
exercises four to five times per week on an upright stationary bicycle and reports no
symptoms. Her medical history includes hyperlipidemia. Her current medications include
atorvastatin 40 mg.

On physical examination her blood pressure is 126/54 mm Hg with a heart rate of 84


KKUH
Collected by:
Dr Hani Abdullah
Dr Fatma Hadi
Dr Salem Boresa
Dr Rami Elesali
Dr Naeif Almagal
1/1/2020
48

bpm. Her lungs are clear to auscultation. The left ventricular (LV) apical impulse is
normal and nondisplaced. A soft, grade 2/4 diastolic decrescendo murmur is present at
the left sternal border. A systolic ejection click is noted. There is no S3 or S4, and no
peripheral edema.

Transthoracic echocardiogram demonstrates an LV ejection fraction of 59% with an


end-systolic dimension of 33 mm and an end-diastolic dimension of 50 mm. The aortic
root diameter is 3.9 cm at the sinuses of Valsalva. There is a BAV with severe AR.

Which of the following is the next best step the treatment of this patient?
A. Metoprolol succinate 25 mg.
B. No additional medications.
C. Enteric coated aspirin 81 mg.
D. Extended release nifedipine 30 mg.
E. Enalapril 5 mg.

In the absence of other indications (e.g., hypertension), vasodilator therapy is not


indicated in asymptomatic patients with AR and normal systolic function. It is a Class I
indication to use vasodilators, preferably dihydropyridine calcium channel blockers,
angiotensin-converting enzyme inhibitors, or angiotensin receptor blockers, to treat
hypertension when present in patients with AR.

For patients with AR, long-term therapy with vasodilators is indicated in those with
severe regurgitation and symptoms or LV dysfunction only when surgical intervention
cannot be performed due to comorbidities (Class IIa).

There is no indication for aspirin or beta-blockers for modification of her risk for eventual
aortic valve replacement. Answer B
Key Point
Surgery remains the mainstay of therapy. Medical therapy is limited to individuals with
chronic aortic regurgitation and hypertension, and it is limited in those who are not
surgical candidates because of comorbidities.

KKUH
Collected by:
Dr Hani Abdullah
Dr Fatma Hadi
Dr Salem Boresa
Dr Rami Elesali
Dr Naeif Almagal
1/1/2020
49

Question 29
29-A 67-year-old male with chronic severe aortic regurgitation (AR) and trileaflet aortic
valve presents to the cardiology clinic for routine outpatient follow-up. He denies any
dyspnea with walking at a moderate pace, mowing his lawn, or other usual activities.
His exam is notable for a II/IV diastolic decrescendo murmur along the left sternal
border. His jugular venous pressure is normal, his lungs are clear, and there is no
peripheral edema. An echocardiogram is obtained.

If present, which of the following echocardiographic findings would support surgical


aortic valve replacement (AVR) for this patient?
A. A left ventricular ejection fraction of 55%.
B. An aortic root of 4.8 cm at the sinuses of Valsalva.
C. A left ventricular end systolic dimension of 52 mm.
D. A left ventricular end diastolic dimension of 61 mm.
E. Holodiastolic flow reversal in the abdominal aorta.

In asymptomatic patients with chronic severe AR, indications for AVR include: a left
ventricular (LV) ejection fraction of <50% (Class I), an LV end systolic dimension of >50
mm (Class IIa), progressive LV enlargement with an LV end diastolic dimension of >65
mm if the patient is a low surgical risk (Class IIb), and if undergoing cardiac surgery for
other indications (Class I). Holodiastolic flow reversal in the abdominal aorta is a highly
specific sign for severe aortic insufficiency, but is not by itself an indication for surgery.
In patients with bicuspid aortic valve who have an indication for AVR, the ascending
aorta should be replaced if its diameter is >4.5 cm. Otherwise, the indication for
ascending aorta replacement is a diameter of >5.5 cm. This patient has a trileaflet aortic
valve. Answer C
Key Point
Left ventricular dysfunction and dilatation can be reversible if valve replacement is
performed in a timely fashion. Therefore, serial follow-up with clinical examinations and
echocardiography is recommended to identify patients who require intervention before
symptoms develop.

KKUH
Collected by:
Dr Hani Abdullah
Dr Fatma Hadi
Dr Salem Boresa
Dr Rami Elesali
Dr Naeif Almagal
1/1/2020
50

Question 30
30- A 48-year-old male with a history of myxomatous valve disease and moderate mitral
regurgitation (MR) on echocardiogram presents today for his annual reassessment. He
has a history of hyperlipidemia and remote endocarditis. He feels well and rides his road
bike most days of the week without limiting symptoms. His exam is notable for a heart
rate 52 bpm, blood pressure of 118/74 mm Hg, and pulse oximetry of 98%. His lungs
are clear without rales. His cardiac exam reveals a soft, late systolic murmur at the
apex. There is no lower extremity edema.

Which of the following would be recommended for this patient?


A. Antibiotics prior to dental cleaning.
B. Aspirin 81 mg daily.
C. Cardiac magnetic resonance imaging.
D. Transesophageal echocardiogram.
E. Losartan 25 mg daily.

Although myxomatous mitral valve disease alone is not an indication for endocarditis
prophylaxis in the current guideline, this patient has a history of prior endocarditis and
therefore should receive antibiotic prophylaxis before dental procedures.

Losartan or other vasodilators are not indicated in MR unless the patient has
hypertension. A transesophageal echocardiogram or magentic resonance can be
helpful to better delineate the severity of the regurgitation and the mechanism.
However, this patient is asymptomatic with only moderate MR and therefore additional
imaging would not change management. Although patients with MR have an increased
risk of developing atrial fibrillation, there is no evidence to support prophylactic aspirin.
He is not in the age range for aspirin prophylaxis for primary prevention.Answer A

Question 31
31- A 48-year-old male with a history of myxomatous valve disease and moderate mitral
regurgitation (MR) on echocardiogram presents today for his annual reassessment. He
has a history of hyperlipidemia and remote endocarditis. He feels well and rides his road
bike most days of the week without limiting symptoms. His exam is notable for a heart
rate 52 bpm, blood pressure of 118/74 mm Hg, and pulse oximetry of 98%. His lungs

KKUH
Collected by:
Dr Hani Abdullah
Dr Fatma Hadi
Dr Salem Boresa
Dr Rami Elesali
Dr Naeif Almagal
1/1/2020
51

are clear without rales. His cardiac exam reveals a soft, late systolic murmur at the
apex. There is no lower extremity edema.

Which of the following would be recommended for this patient?


A. Antibiotics prior to dental cleaning.
B. Aspirin 81 mg daily.
C. Cardiac magnetic resonance imaging.
D. Transesophageal echocardiogram.
E. Losartan 25 mg daily.

Although myxomatous mitral valve disease alone is not an indication for endocarditis
prophylaxis in the current guideline, this patient has a history of prior endocarditis and
therefore should receive antibiotic prophylaxis before dental procedures.

Losartan or other vasodilators are not indicated in MR unless the patient has
hypertension. A transesophageal echocardiogram or magentic resonance can be
helpful to better delineate the severity of the regurgitation and the mechanism.
However, this patient is asymptomatic with only moderate MR and therefore additional
imaging would not change management. Although patients with MR have an increased
risk of developing atrial fibrillation, there is no evidence to support prophylactic aspirin.
He is not in the age range for aspirin prophylaxis for primary prevention. Answer A

Question 32
32- A 44-year-old male with a history of bicuspid aortic valve (BAV) comes in to your
office for initial evaluation. He plays basketball regularly and reports no cardiovascular
symptoms. His family history is significant for aortic dissection and repair in his father.
On exam his blood pressure is 132/72 mm Hg, heart rate is 68 bpm, and body mass
index is 26 kg/m2. He has a 3/6 mid-to-late peaking systolic murmur heard best at the
base. He undergoes an echocardiogram which reveals normal biventricular size and
function. His peak transaortic velocity is 3.4 m/sec, peak transaortic gradient is 46 mm

KKUH
Collected by:
Dr Hani Abdullah
Dr Fatma Hadi
Dr Salem Boresa
Dr Rami Elesali
Dr Naeif Almagal
1/1/2020
52

Hg, and mean trans aortic gradient is 29 mm Hg. His calculated aortic valve area is 1.1
cm2. His ascending aorta measures 5.1 cm.

Which of the following is the next most appropriate step in the care of this patient?
A. Transesophageal echocardiogram.
B. Cardiac magnetic resonance imaging.
C. Repeat the echocardiogram in 1 year.
D. Exercise stress testing.
E. Surgical referral.

In this patient with BAV and aortopathy, consideration of surgical repair should occur
when the aortic size is >5.5 cm, there is rapidly increasing size (>0.5 cm/year), or as in
this patient, a family history of dissection (Class IIa). If the transthoracic imaging quality
is adequate, imaging with magnetic resonance or transesophageal imaging is not
indicated. If the patient does not elect to undergo surgery, then repeating the
echocardiogram in 1 year is indicated. Given his high level of activity, exercise stress
testing is not likely to add additional information about his functional status. Answer E

Question 33
33- A 70-year-old female presents for a follow-up office visit following recent cardiac
testing for fatigue, abdominal distention, and lower extremity swelling. She has a history
of paroxysmal atrial fibrillation and sick sinus syndrome requiring a dual chamber
pacemaker. On exam, she has jugular venous distension, normal heart sounds, no
murmur, abdominal distention with a palpable free liver edge, and significant lower
extremity swelling.

A right heart catheterization was performed revealing a right atrial (RA) pressure of 12
mm Hg (v waves 35 mm Hg), right ventricular (RV) pressure of 35/15 mm Hg,
pulmonary artery pressure of 35/12 mm Hg, and pulmonary capillary wedge pressure of
12 mm Hg.

KKUH
Collected by:
Dr Hani Abdullah
Dr Fatma Hadi
Dr Salem Boresa
Dr Rami Elesali
Dr Naeif Almagal
1/1/2020
53

A transthoracic echocardiogram revealed severe eccentric tricuspid regurgitation (TR)


with holosystolic flow reversal in the hepatic veins. Tricuspid valve leaflets were not
thickened and without apical displacement. Anterior and posterior leaflets exhibited
normal motion. Restricted septal leaflet motion was present. RV basal diastolic diameter
was 45 mm.

What is the most likely cause of this patient's TR?


A. Carcinoid syndrome.
B. Ebstein's anomaly.
C. Rheumatic heart disease.
D. Pacemaker lead.
E. Pulmonary arterial hypertension.

This patient has symptomatic severe tricuspid regurgitation (TR) with evidence of right
heart failure on examination. The etiology of severe TR can typically be established
using clinical history and echocardiographic findings. Though not performed for this
patient, transesophageal echocardiography can help to determine the cause of severe
TR. Primary disorders causing TR include rheumatic heart disease, prolapse, congenital
disease (Ebstein's), infective endocarditis, radiation, carcinoid syndrome, blunt chest
wall trauma, RV endomyocardial biopsy-related trauma, and intra-annular RV
pacemaker or implantable cardioverter defibrillator leads. Approximately 80% of cases
of significant TR are functional in nature and related to tricuspid annular dilation and
leaflet tethering in the setting of RV remodeling due to pressure and/or volume
overload.

For this patient, the right heart catheterization reveals significantly elevated RA
pressures with elevated v waves in the absence of pulmonary hypertension. The valve
leaflets are not thickened which excludes carcinoid and rheumatic disease. Rheumatic
tricuspid valve disease typically includes diffuse leaflet thickening with restriction of
opening due to commissural fusion, chordal shortening, and calcification with
characteristic diastolic doming. In carcinoid syndrome, liver metastases produce 5-
hydroxyindoleacetic acid which causes a unique echocardiographic appearance of the
RV including leaflets that are short, thick, and with systolic and diastolic restriction. The
patient does not have echocardiographic characteristics of Ebstein's anomaly, a
congenital disease characterized by apical displacement of the tricuspid valve septal
and posterior leaflets from the atrioventricular ring. The septal and posterior leaflets are
typically adherent to the underlying myocardium with an apical displacement of ≥8
mm/m2 compared with the mitral valve annulus.

Thus, the most likely cause of the patient's severe TR is restriction caused by the
patient's pacemaker lead. Endocardial leads can impair the structure and function of the
tricuspid valve. Injury can also occur during implantation or extraction. Chronic
interaction between the endocardial leads and tricuspid valve leaflets and/or chords can
KKUH
Collected by:
Dr Hani Abdullah
Dr Fatma Hadi
Dr Salem Boresa
Dr Rami Elesali
Dr Naeif Almagal
1/1/2020
54

result in inflammation and fibrosis leading to entrapment of the lead. The resulting TR
will typically exhibit eccentric rather than a central trajectory, with the septal leaflet being
the most common leaflet to be entrapped. Answer D
Key Point
The main cause of tricuspid regurgitation (TR) is secondary or secondary TR due to
annulus dilation.

Question 34
34- A 65-year-old male presents to your office for routine follow-up. He has a past
medical history of symptomatic severe aortic stenosis secondary to bicuspid aortic valve
(AV) disease. Approximately 1 year ago, he underwent surgical AV replacement using a
21 mm bioprosthetic valve. His postoperative course was uneventful.

He now presents to your clinic 1 year later and reports exertional fatigue with
ambulating one block that has been present since surgery. On exam, he has a normal
first and second heart sound with a 3/6 mid-late peaking systolic murmur that is best
heard at the right upper sternal border. Body surface area is 2.18 m2. His remaining
physical examination is unremarkable.

The following AV Doppler measurements were obtained at 2 and 12 months: 2 month


left ventricular outflow tract (LVOT) diameter 20 mm; LVOT velocity time integral (VTI)
22 cm; AV VTI 65 cm; effective orifice area (EOA) 1.06 cm2; AV mean gradient 28 mm
Hg; acceleration time 92 msec; 12-month LVOT diameter 20 mm; LVOT VTI 25 cm; AV
VTI 70 cm; EOA 1.12 cm2; AV mean gradient 36 mm Hg; acceleration time 97 msec.

Which of the following is the most likely cause for this patient's elevated transaortic
gradients?
A. Pannus formation.
B. Patient-prosthesis mismatch.
C. Hemolytic anemia.
KKUH
Collected by:
Dr Hani Abdullah
Dr Fatma Hadi
Dr Salem Boresa
Dr Rami Elesali
Dr Naeif Almagal
1/1/2020
55

D. Pressure recovery.
E. Valve thrombosis.

The patient has elevated gradients across his AV prosthesis, with a higher than
expected mean gradient and a smaller than expected EOA. Thus, it is important to
consider prosthetic valve obstruction as a cause for elevated prosthetic valve gradients.
Obstruction can be functional (i.e., patient-prosthesis mismatch [PPM]) or pathologic
(e.g., thrombus or pannus formation). It is important to calculate the indexed EOA by
dividing the EOA by the body surface area. The indexed EOA at 2 and 12 months are
0.49 and 0.52 cm2/m2, respectively. When the indexed EOA is ≤0.65 cm2/m2, it suggests
severe PPM. Additionally, an acceleration time is usually ≤100 msec in PPM, whereas it
is >100 msec with pathologic obstruction. The Doppler velocity index ([DVI]; i.e., LVOT
VTI/AV VTI) is 0.34 and 0.36 at 2 and 12 months respectively. PPM will typically have a
DVI >0.25, whereas in pathological obstruction it will be <0.25. Additionally, the Doppler
gradients are stable over time without a significant change, making pathological
obstruction less likely. Taken together, the correct answer is PPM, which is the most
common cause of elevated gradients across a surgical bioprosthetic valve.

Hemolytic anemia can can cause increased cardiac output and therefore increased
gradients, however it will not affect the DVI or indexed EOA. Pressure recovery
phenomenon causing elevated gradients is more common in mechanical prostheses.
Answer B
Key Point
Patient prosthesis mismatch (PPM) is the most frequent cause of high transprosthetic
gradients following aortic valve replacement or mitral valve replacement. The differential
diagnosis between PPM and acquired prosthetic valve stenosis can be made by: 1)
assessing leaflet morphology and mobility; 2) comparing measured echocardiographic
parameters of prosthetic valve function to normal reference values; and 3) assessing
serial changes in the echocardiographic parameters during follow-up.

KKUH
Collected by:
Dr Hani Abdullah
Dr Fatma Hadi
Dr Salem Boresa
Dr Rami Elesali
Dr Naeif Almagal
1/1/2020
56

Question 35
35-A 58-year-old female presents to your office with a 6-month history of palpitations
and dyspnea on exertion. She has no medical problems and takes no medications.

Her examination reveals a body mass index of 30 kg/m2, a heart rate of 68 bpm, and a
blood pressure 160/80 mm Hg. Her jugular venous pressure is 6 cm H20. Her cardiac
exam reveals a systolic murmur with a single component S2.

What test is most likely to elucidate the diagnosis?


A. Injection of agitated saline contrast.
B. Chest X-ray.
C. Electrocardiogram.
D. Doppler tricuspid regurgitation velocity.
E. Doppler aortic velocity across the aortic valve.

This patient presents with dyspnea on exertion as well as a systolic murmur, but the
telltale finding on examination is the single S2. A single S2 can be caused by pulmonic
stenosis, severe aortic stenosis, congenital absence of the pulmonic valve, or in
patients in whom cardiac auscultation is hindered by body habitus, emphysema, or
pericardial effusion. Doppler velocity across the aortic valve would assess for aortic
stenosis and thus is the correct answer.

An electrocardiogram (ECG) would show a right or left bundle branch block, but the
former would result in a widely split S2 and the latter a paradoxically split S2. Thus, ECG
is not the correct answer.

Agitated saline contrast would assess for an atrial septal defect which would cause a
fixed split S2.

Doppler of the tricuspid regurgitant jet would assess the estimated right ventricular
systolic pressure, an indicator of pulmonary hypertension, which would cause a fixed
split S2 with increased intensity of P2.

A chest X-ray may show rib notching as seen in aortic coarctation. This would not result
in an abnormality of the second heart sound and thus is not the correct answer. Answer
E
Key Point
A single S2 may be present in severe pulmonary stenosis or aortic stenosis,
transposition of the great arteries, or pulmonary hypertension.

KKUH
Collected by:
Dr Hani Abdullah
Dr Fatma Hadi
Dr Salem Boresa
Dr Rami Elesali
Dr Naeif Almagal
1/1/2020
57

Question 36
36-A 67-year-old female presents for follow-up for her valvular heart disease. Her
history is significant for hypertension, hyperlipidemia, and aortic insufficiency. Her
medications include hydrochlorothiazide 25 mg daily and aspirin 81 mg. Her exam is
significant for a blood pressure (BP) of 160/90 mm Hg, a heart rate of 72 bpm, a 2/4
holodiastolic murmur heard best at the apex, and a soft 1/6 holosystolic murmur heard
best at the apex. Her creatinine is 0.8 mg/dl. She undergoes echocardiography which
revealed normal left ventricular cavity size and function, moderate aortic insufficiency,
and mild mitral regurgitation.

Which of the following is most appropriate in the care of this patient?


A. The addition of verapamil extended release 120 mg daily.
B. Increase hydrochlorothiazide to 50 mg daily.
C. The addition of metoprolol succinate 50 mg daily.
D. The addition of losartan 25 mg daily.
E. The addition of hydralazine 10 mg three times daily.

This patient has stage B aortic regurgitation (AR) and hypertension. Treatment of
hypertension (systolic BP >140 mm Hg) is recommended in patients with chronic AR
(stages B and C), preferably with dihydropyridine calcium channel blockers or
angiotensin-converting enzyme inhibitors/angiotensin-receptor blockers (e.g., losartan;
Class 1, Level of Evidence B). Beta-blockers may be less effective because the
reduction in heart rate is associated with an even higher stroke volume, which
contributes to the elevated systolic pressure in patients with chronic severe AR. While
each of the other answer options may help with BP control, they are not the next best
choice of antihypertensive therapy.
Answer D
Key Point
Surgery remains the mainstay of therapy. Medical therapy is limited to individuals with
chronic aortic regurgitation and hypertension, and it is limited in those who are not
surgical candidates because of comorbidities.

KKUH
Collected by:
Dr Hani Abdullah
Dr Fatma Hadi
Dr Salem Boresa
Dr Rami Elesali
Dr Naeif Almagal
1/1/2020
58

Question 37
37- A 50-year-old male is hospitalized following an inferior ST-segment elevation
myocardial infarction (MI). He had late revascularization of an occluded, dominant right
coronary artery. On day 3 he developed acute breathlessness. His vital signs are
noteworthy for a heart rate of 110 bpm, a blood pressure of 102/80 mm Hg, and a
respiratory rate of 28 breaths per minute with an oxygen saturation of 90% on high flow
oxygen. His physical exam reveals wet rales bilaterally. He is tachycardic, but cardiac
auscultation reveals no murmur. His 12-lead electrocardiogram shows sinus tachycardia
with inferior Q waves and <1 mm ST-segment depression.

What is the most appropriate next step in the management of this patient?
A. Procalcitonin.
B. B-type natriuretic peptide.
C. Coronary angiography.
D. Pulmonary artery catheter placement.
E. Echocardiogram.

This patient is in extremis from acute mitral valve regurgitation (MR), possibly as a
consequence of papillary muscle rupture. In this scenario, inferior MR is likely
associated with posteromedial papillary muscle rupture. Acute severe MR may produce
a brief systolic murmur or no murmur at all. Rapid diagnosis is essential. A delay in the
time to surgery appears to increase the risk of further myocardial injury, organ failure,
and death.

To confirm the diagnosis, an urgent echocardiogram is needed. Transthoracic


echocardiography may be sufficient, however eccentric MR may be under-appreciated
on transthoracic imaging, in which case transesophageal imaging would be needed.

The other answer options would fail to diagnose the problem in a timely manner.
Pulmonary artery catheter placement might reveal an elevated wedge pressure tracing
with a V-wave, but this is supportive data as opposed to direct confirmation of MR. One
might consider right heart catheterization data useful in the setting of an ischemic
ventricular septal rupture (VSR), where an oxygenation step-up would be noted,
however the clinical vignette doesn't include features of a VSR such as a loud murmur;
and in the case of VSR, pulmonary edema would be an uncommon feature. Coronary
angiography would be warranted if there was concern for reinfarction, but the clinical
vignette is not supportive of reinfarction. B-type natriuretic peptide and procalcitonin
levels are not likely to be helpful in the acute management of a patient with a suspected
mechanical complication of MI. Answer E
Key Point
The murmur of acute mitral regurgitation (MR) differs from that of chronic MR because

KKUH
Collected by:
Dr Hani Abdullah
Dr Fatma Hadi
Dr Salem Boresa
Dr Rami Elesali
Dr Naeif Almagal
1/1/2020
59

of the rapid rise in left atrium pressures in the acute setting. The acute MR murmur may
be short.

Question 38
38-A 45-year-old female with history of a bileaflet mechanical aortic valve replacement
for bicuspid aortic valve disease 10 years ago is admitted with complaints of dyspnea
on exertion. She has had sporadic medical care over the past year. She reports
occasional recent subjective fevers. Her past medical history is otherwise
unremarkable. On examination she is afebrile with a heart rate of 90 bpm and a blood
pressure of 110/75 mm Hg. She has clear lung fields. On cardiac auscultation there is a
soft systolic murmur.

Which of the following would be the most likely finding on this patient's
echocardiogram?
A. An aortic valve acceleration time 70 msec.
B. A dimensionless index of 0.44.
C. A left ventricular outflow tract gradient of 40 mm Hg.
D. A peak aortic velocity 4.1 m/sec.
E. An effective orifice area 1.5 cm2.

This patient has prosthetic aortic valve stenosis in the setting of sporadic medical care,
suspicious for valve thrombosis from inadequate anticoagulation. Transthoracic
echocardiography is the first-line test for diagnosing prosthetic valve dysfunction.
Expected findings would be an elevated transvalvular velocity and gradient; a prolonged
(>100 msec) acceleration time; a reduced effective orifice area (<1 cm2); and a reduced
dimensionless index (<0.3).
A left ventricular outflow tract gradient would not be expected to occur in this setting and
would also be expected to cause a dynamic murmur. Answer D

KKUH
Collected by:
Dr Hani Abdullah
Dr Fatma Hadi
Dr Salem Boresa
Dr Rami Elesali
Dr Naeif Almagal
1/1/2020
60

Question 39
39-An 88-year-old female presents to clinic for progressive dyspnea on exertion and
lower extremity edema. Her past history includes osteoarthritis and hypertension. She
takes aspirin 81 mg daily, metoprolol succinate 25 mg daily, amlodipine 5 mg daily, and
naproxen as needed.

On exam, her blood pressure is 130/60 mm Hg, pulse is 70 bpm, and respirations are
16 breaths per minute. Her jugular venous pressure is 10 cm H 20. There are bibasilar
crackles. She has a late-peaking, harsh systolic murmur along the right upper sternal
border with a single S2. There is 2+ bilateral lower extremity edema.

Her echocardiogram shows a severely calcified aortic valve with reduced leaflet
excursion. Peak velocity across the valve is 3.3 m/sec with a mean gradient of 28 mm
Hg. The calculated valve area was 0.8 cm2 and dimensionless index is 0.22. The left
ventricle (LV) is mildly dilated with a left ventricular ejection fraction (LVEF) of 25-30%
and global hypokinesis.

Which of the following is the most appropriate next step in the management of this
patient?
A. Exercise myocardial perfusion scan.
B. Right and left heart catheterization.
C. Cardiac magnetic resonance imaging.
D. Transesophageal echocardiography.
E. Dobutamine stress echocardiography.

Most patients with severe aortic stenosis (AS) present with a high transvalvular gradient
and velocity. However, a subset present with severe AS despite a low gradient and
velocity due either to concurrent LV systolic dysfunction (LVEF of <50%) or a low
transaortic stroke volume with preserved LV systolic function.

Outcomes in severe low-flow, low-gradient AS are improved with aortic valve


replacement (AVR) compared with medical therapy particularly when contractile reserve
is present. The American Society of Echocardiography/European Association of
Echocardiography recommendations for clinical practice defines severe AS on
dobutamine stress testing as a maximum velocity of >4.0 m/sec with a valve area of 1.0
cm2 at any point during the test protocol, with a maximum dobutamine dose of 20
mcg/kg per minute. On the basis of outcome data in several prospective nonrandomized
studies, AVR is reasonable in these patients (Class IIa). LVEF typically increases by 10
KKUH
Collected by:
Dr Hani Abdullah
Dr Fatma Hadi
Dr Salem Boresa
Dr Rami Elesali
Dr Naeif Almagal
1/1/2020
61

LVEF units and may return to normal if afterload mismatch was the cause of the LV
systolic dysfunction. Some patients without contractile reserve may also benefit from
AVR, but decisions in these high-risk patients must be individualized because there are
no data indicating who will have a better outcome with surgery.

Exercise stress testing is contraindicated in patients with severe symptomatic AS and


nuclear perfusion imaging would not be expected to add useful information in this
setting.

Right and left heart catheterization are invasive procedures and generally reserved for
cases with indeterminate noninvasive test results.

The role of cardiac magnetic resonance imaging for low-flow, low-gradient AS has not
been established.

Transesophageal echocardiography would not be expected to add additional


information. Answer E

Key Point
With low-flow, low-gradient aortic stenosis (AS) and a reduced left ventricular ejection
fraction (<50%), severe AS is defined as a velocity of 4 m/sec and a valve area of ≤1.0
cm2, at any flow rate, on low-dose dobutamine stress echocardiogram.

Question 40
40- A 51-year-old male is referred for evaluation of a chronic murmur. At 20 years of
age, he underwent surgical repair of bicuspid aortic valve (BAV) stenosis. The patient
has been followed intermittently and has no other significant medical history. He is
physically active and asymptomatic, regularly running 5 miles without limitation.

On physical examination his heart rate is regular at 60 bpm and he has a blood
pressure of 135/55 mm Hg. His lungs are clear to auscultation. Left ventricular (LV)
apical impulse is enlarged and laterally displaced to the anterior axillary line. A grade
KKUH
Collected by:
Dr Hani Abdullah
Dr Fatma Hadi
Dr Salem Boresa
Dr Rami Elesali
Dr Naeif Almagal
1/1/2020
62

2/6 early-peaking systolic murmur and a decrescendo grade 2/4 diastolic murmur both
are present along the left sternal border. The S2 is soft. There is no S3or S4. Peripheral
examination demonstrates a pulsatile uvula and a collapsing water hammer radial
pulse.
The patient undergoes a transthoracic echocardiogram that demonstrates BAV. LV
end-diastolic dimension is 6.5 cm, LV end-systolic dimension is 4.2 cm, and LV ejection
fraction is 57%. There is severe aortic regurgitation by color Doppler. Atrioventricular
(AV) regurgitant volume is 70 ml, AV regurgitant fraction is 60%, and aortic root
diameter is 4.2 cm.

Which of the following is the next best step in the management of this patient?
A. Symptom-limited exercise stress test.
B. Serial echocardiography.
C. Surgical aortic valve replacement.
D. Transcatheter aortic valve replacement.

This patient has asymptomatic severe aortic regurgitation (AR). Surgical aortic valve
replacement (AVR) is a Class I indication if the LV ejection fraction is <50% or if the
patient is undergoing cardiac surgery for another indication. For an asymptomatic
patient with severe AR, surgical AVR is a Class IIa recommendation when there is an
end-systolic dimension of >5.0 cm and a Class IIb indication when there is an LV end-
diastolic dimension of >6.5 cm (Figure 1).

As intervention is not currently indicated for this patient, the most appropriate
recommendation is to repeat the echocardiogram in 6-12 months.

A symptom-limited exercise stress test is reasonable to clarify functional capacity in


patients with equivocal symptoms, but is not indicated in a patient who can run 5 miles
without limitations.

Transcatheter AVR is not indicated in this asymptomatic patient and is also not
indicated for management of AR. Answer B

KKUH
Collected by:
Dr Hani Abdullah
Dr Fatma Hadi
Dr Salem Boresa
Dr Rami Elesali
Dr Naeif Almagal
1/1/2020
63

Key Point
Left ventricular dysfunction and dilatation can be reversible if valve replacement is
performed in a timely fashion. Therefore, serial follow-up with clinical examinations and
echocardiography is recommended to identify patients who require intervention before
symptoms develop.

KKUH
Collected by:
Dr Hani Abdullah
Dr Fatma Hadi
Dr Salem Boresa
Dr Rami Elesali
Dr Naeif Almagal
1/1/2020
64

Question 41
41- A 66-year-old female is postoperative Day 2 from cardiac surgery for symptomatic
mitral regurgitation. A 31 mm mechanical bileaflet mitral prosthesis was placed and a
tricuspid annuloplasty was performed. She had postoperative atrial fibrillation which
spontaneously resolved within 24 hours. Her medical history is significant only for
hypertension. A preoperative coronary angiogram demonstrated normal coronary
arteries.

Which of the following is the most appropriate long-term antithrombotic therapy for this
patient?
A. Warfarin with an international normalized ratio goal of 3.0-4.0.
B. Warfarin with an international normalized ratio goal of 2.5-3.5.
C. Aspirin 81 mg daily and warfarin with an international normalized ratio goal of 2.0-3.0.
D. Aspirin 81 mg daily and warfarin with an international normalized ratio goal of 2.5-3.5.

For patients with a mechanical mitral valve prosthesis, warfarin with an with an
international normalized ratio (INR) goal of 3 (2.5-3.5) in addition to aspirin 75-100 mg
daily is a Class I recommendation.

Higher INR goals may be considered in patients who have thromboembolic events while
anticoagulated within this INR range. This patient has not had a thromboembolic event
therefore a higher INR goal would not be indicated.

Warfarin alone is not adequate. Randomized trials have demonstrated a reduction of


thromboembolic events with the addition of aspirin (75-100 mg daily) to warfarin
anticoagulation in patients with mechanical valves.
Answer D

KKUH
Collected by:
Dr Hani Abdullah
Dr Fatma Hadi
Dr Salem Boresa
Dr Rami Elesali
Dr Naeif Almagal
1/1/2020
65

Question 42
42- You are asked to consult on a 70-year-old male with shortness of breath and a
murmur. His past medical history includes hypertension, hypercholesterolemia, and
arthritis. His current medications include aspirin 81 mg, atenolol 50 mg, atorvastatin 80
mg, and naproxen 375 mg as needed. His vital signs are a pulse rate of 70 bpm, blood
pressure of 110/76 mm Hg, and his respiration rate is 16 breaths per minute. His
physical exam is remarkable for jugular venous distension to 8 cm above the sternal
angle, right basilar crackles, a soft holosytolic murmur, and 1+ bilateral ankle edema.

An echocardiogram is performed (Videos 1, 2, 3).

What is the most likely etiology of this patient's mitral valve disease?
Long axis view : RWMA at inferolateral wall
Long axis view, with color : Severe MR posterior directed
Short axis view : RWMA at inferolateral wall
A. Coronary atherosclerosis.
B. Postinfectious inflammation.
C. Congenital deformation.
D. Dystrophic calcification.
E. Myxoid degeneration.

This patient has functional or secondary mitral regurgitation (MR) caused by


inferolateral hypokinesis of the left ventricle with tethering of the lateral papillary muscle;
the most likely cause is right coronary or left circumflex territory coronary artery disease.
MR can be classified according to leaflet motion, which gives clues to the differential
diagnosis of the cause (Figure 1).

Type I MR involves normal leaflet motion. This can be caused by endocarditis with
perforation of a leaflet or by a cleft mitral valve (congenital).

Type II MR is due to excessive leaflet motion, either flail or prolapse, due to fibroelastic
deficiency or mitral valve prolapse syndrome.

Type III MR is due to restricted leaflet motion. Type IIIa MR is due to restricted diastolic
and systolic motion and may be caused by mitral annular calcification or rheumatic
disease (post-streptococcal infection). Type IIIb MR is due to restricted systolic motion
and is caused by left ventricular dysfunction, as in this case. Answer A

KKUH
Collected by:
Dr Hani Abdullah
Dr Fatma Hadi
Dr Salem Boresa
Dr Rami Elesali
Dr Naeif Almagal
1/1/2020
66

Key Point
Anatomic or functional abnormalities of the mitral annulus, leaflets, subvalvular
apparatus, or left ventricle may result in mitral regurgitation.

Question 43
43- A 52-year-old male with no prior medical history presents to the emergency
department with 2 weeks of progressive dyspnea, lower extremity edema, and
orthopnea. Exam is notable for a temperature 39.1 degrees Celsius, an irregular heart
rate of 125 bmp, a blood pressure of 110/85 mm Hg, a respiratory rate 33 breaths per
minute, and a pulse oximetry of 85% on room air. His echocardiogram shows a left
ventricular ejection fraction (LVEF) of 72% and an 0.8 cm mass on the anterior mitral
valve leaflet with severe eccentric mitral regurgitation due to leaflet perforation. Cultures
are obtained and he is started on intravenous antibiotics and furosemide. He is taken to
the operating room where a 25 mm mechanical mitral valve is placed. His postoperative
course is complicated by acute kidney injury and anemia requiring blood transfusions.

He is now 2 months postoperative and is being seen in your office. He denies shortness
of breath, but does admit to fatigue. On exam his heart rate is 78 bpm and blood
KKUH
Collected by:
Dr Hani Abdullah
Dr Fatma Hadi
Dr Salem Boresa
Dr Rami Elesali
Dr Naeif Almagal
1/1/2020
67

pressure is 125/72 mm Hg. A crisp mechanical S1 is heard. His lungs are clear and
there is no edema. His current medications include aspirin 81 mg daily, warfarin, and
metoprolol tartrate 50 mg twice a day. His laboratory results are notable for hemoglobin
8.8 g/dl, creatinine 1.1 mg/dl, and international normalized ratio 3.5 (INR; range of 1.7-
4.2 over the past month).

What is the next best step in management of this patient?


A. Switch to metoprolol succinate 100 mg daily.
B. Stop aspirin.
C. Perform transthoracic echocardiogram.
D. Switch to rivaroxaban.
E. Change INR target to 2-2.5.

An echocardiographic examination performed 6 weeks to 3 months after valve


implantation is essential to establish a baseline for comparison should complications or
deterioration occur later (Class I recommendation, Level of Evidence B). Despite the
anemia, the aspirin should not be stopped as it is indicated along with a vitamin K
antagonist in patients with mechanical prosthetic valves. While metoprolol succinate is
preferred in patients with heart failure with reduced ejection fraction, there is no clear
benefit in patients following valve replacement with normal LVEF. Anticoagulant therapy
with oral direct thrombin or anti-Xa agents should not be used in patients with
mechanical valve prostheses (Class III recommendation), therefore switching to
rivaroxaban is incorrect. Answer C
Key Point
Periodic clinical and echocardiographic evaluation of patients with a prosthetic valve is
essential for early detection of prosthetic valve dysfunction. Careful medical
management of patients with prosthetic valves includes careful control of antithrombotic
therapy and prescription of infective endocarditis prophylaxis.

KKUH
Collected by:
Dr Hani Abdullah
Dr Fatma Hadi
Dr Salem Boresa
Dr Rami Elesali
Dr Naeif Almagal
1/1/2020
68

Question 44
44- A 56-year-old female with a history of mitral valve prolapse (MVP) and a recent
dental procedure presents to the emergency department with fevers and shortness of
breath. An echocardiogram one year prior showed bileaflet MVP with trace mitral
regurgitation (MR). She has no other medical problems and takes no medications. On
examination her temperature was 102.4 F, heart rate was 110 bpm, and blood pressure
was 102/70 mm Hg. She is diaphoretic and appears to be in mild distress. Her jugular
venous pressure is 6 cm H20. Her lungs have bibasilar fine inspiratory crackles. Her
heart is regular with a soft systolic murmur and soft S3 gallop. Her abdomen is soft and
nontender; her extremities are warm without edema. Her echocardiogram shows an
ejection fraction of 65% and thickening of the mitral valve leaflets with moderate MR.

Development of which the following would be an indication to perform a


transesophageal echocardiogram?
A. Atrial fibrillation.
B. Streptococcus bovis bacteremia.
C. Worsening pulmonary edema.
D. Persistent fever after 48 hours.
E. Acute deep vein thrombosis of the femoral vein.

Transthoracic and/or transesophageal echocardiograpy (TEE) are recommended for the


re-evaluation of patients with infective endocarditis (IE) who have a change in clinical
signs or symptoms (e.g., new murmur, embolism, persistent fever, heart failure [HF],
abscess, or atrioventricular heart block) and in patients at a high risk of complications
(e.g., extensive infected tissue/large vegetation on initial echocardiogram
or staphylococcal, enterococcal, fungal infections).

HF, perivalvular extension, and embolic events represent three of the most frequent and
severe complications of IE. They are also the three main indications for early surgery,
which is performed in almost 50% of cases. If signs or symptoms consistent with any of
these complications exist, there should be a very low threshold for repeat imaging in
these patients. Conversely, in the absence of clinical deterioration or new
signs/symptoms, routine follow-up echocardiography is probably of only limited clinical
utility.

In this patient, worsening HF is the indication to perform TEE to assess for progressive
valvular destruction that may be an indication for urgent surgery. Perivalvular abscesses
can extend into adjacent cardiac conduction tissues, leading to heart block. Involvement
of the conducting system is most common in the setting of aortic valve infection,
especially when there is involvement of the valve ring between the right and
noncoronary cusp; this anatomic site overlies the intraventricular septum that contains
the proximal ventricular conduction system.

KKUH
Collected by:
Dr Hani Abdullah
Dr Fatma Hadi
Dr Salem Boresa
Dr Rami Elesali
Dr Naeif Almagal
1/1/2020
69

However, atrial fibrillation would not be considered a sign of a perivalvular abscess and
thus not an indication for repeat imaging. Fever associated with IE should resolve after
3-5 days of antibiotic therapy. A fever noted after 48 hours in the absence of other
symptoms or signs would not be considered a failure of therapy and thus further
imaging would not be indicated. Streptococcus bovis (S. bovis) is a common cause of
IE, but unlike IE caused by Staphylococcus species, enterococcus, and fungal
species, S. bovis IE usually responds to antibiotic therapy so the growth of S. bovis from
blood cultures alone would not be a reason to perform repeat imaging. Septic emboli
can occur with IE resulting in stroke, renal or splenic infarcts, ischemia of the
extremities, myocardial infarction or, in the case of right-sided IE, pulmonary embolism.
However, deep vein thrombosis (DVT) would not be a form of septic emboli and thus
the presence of an acute DVT would not warrant further imaging. Answer C
Key Point
Transesophageal echocardiography is indicated for diagnosis and evaluation of
vegetation size, abscess formation, fistula formation, leaflet perforation, or prosthetic
valve dehiscence.

Question 45
45- A 60-year-old male presents for evaluation of a murmur. He exercises by walking
his dog two miles daily without limitations. He has a history of hypertension and
hyperlipidemia. His medications include amlodipine 5 mg daily and atorvastatin 20 mg
daily.

On examination, his heart rate is 70 bpm, blood pressure is 128/80 mm Hg, and jugular
venous pressure is 4 cm H20. His lungs are clear. His cardiac exam shows a soft
systolic ejection murmur radiating to the carotid arteries. His extremities have no
edema.

His echocardiogram shows an ejection fraction of 60% and aortic stenosis (AS) with a
peak velocity of 2.3 m/sec, mean gradient of 13 mm Hg, and a valve area of 1.8 cm2.

In the absence of new symptoms, what is the appropriate interval for a repeat
echocardiogram in this patient?
KKUH
Collected by:
Dr Hani Abdullah
Dr Fatma Hadi
Dr Salem Boresa
Dr Rami Elesali
Dr Naeif Almagal
1/1/2020
70

A. 2 years.
B. 6 months.
C. 3 years.
D. 6 years.
E. 1 year.

The timing of periodic clinical evaluation of patients with severe asymptomatic AS


depends on comorbidities and patient-specific factors. Transesophageal
echocardiogram for re-evaluation of asymptomatic patients with AS with normal left
ventricular systolic function who have no change in signs or symptoms is performed at
intervals of 6 months to 1 year when aortic velocity is 4.0 m/sec (stage C), 1-2 years
when aortic velocity is 3.0-3.9 m/sec (stage B), and 3-5 years when aortic velocity is
2.0-2.9 m/sec (stage B; Table 1).

Valvular AS is a progressive disease, and an increase in hemodynamic severity is


inevitable once even mild AS is present. The rate of progression of the stenotic lesion
has been estimated in a variety of invasive and noninvasive studies. When severe AS is
present (aortic velocity of 4.0 m/sec), the rate of progression to symptoms is high with
an event-free survival of only 30-50% at 2 years. Therefore, patients with asymptomatic
severe AS require frequent monitoring for progressive disease because symptom onset
may be insidious and not recognized by the patient.

Once even moderate AS is present (aortic velocity between 3.0-3.9 m/sec), the average
rate of progression is an increase in velocity of 0.3 m/sec per year, an increase in mean
pressure gradient of 7 mm Hg per year, and a decrease in valve area of 0.1 cm 2 per
year. There is marked individual variability in the rate of hemodynamic change.
Progression of AS can be more rapid in older patients and in those with more severe
leaflet calcification. Because it is not possible to predict the exact rate of progression in
an individual patient, regular clinical and echocardiographic follow-up is mandatory in all
patients with asymptomatic mild-to-moderate AS.

In his patient with mild (Stage B) AS, a repeat echocardiogram in 3-5 years is
appropriate. Answer C

KKUH
Collected by:
Dr Hani Abdullah
Dr Fatma Hadi
Dr Salem Boresa
Dr Rami Elesali
Dr Naeif Almagal
1/1/2020
71

Question 46
46- You are seeing an 80-year-old female for shortness of breath. She reports dyspnea
with usual activites such as walking in the grocery store. She has hypertension, stage 3
chronic kidney disease, hyperlipidemia, and osteoporosis. Her current medications
include lisinopril, simvastatin, calcium plus vitamin D, and aspirin. Her cardiac exam
reveals a soft systolic ejection murmur with a normally split S2. She has arthritic
changes of the joints of both of her hands. Her exam is otherwise unremarkable.

An echocardiogram shows a mean transmitral gradient 6 mm Hg at heart rate of 70


bpm.

Which of the following most likely represents the appearance of this patient's mitral
valve?

KKUH
Collected by:
Dr Hani Abdullah
Dr Fatma Hadi
Dr Salem Boresa
Dr Rami Elesali
Dr Naeif Almagal
1/1/2020
72

KKUH
Collected by:
Dr Hani Abdullah
Dr Fatma Hadi
Dr Salem Boresa
Dr Rami Elesali
Dr Naeif Almagal
1/1/2020
73

KKUH
Collected by:
Dr Hani Abdullah
Dr Fatma Hadi
Dr Salem Boresa
Dr Rami Elesali
Dr Naeif Almagal
1/1/2020
74

KKUH
Collected by:
Dr Hani Abdullah
Dr Fatma Hadi
Dr Salem Boresa
Dr Rami Elesali
Dr Naeif Almagal
1/1/2020
75

Although the vast majority of mitral stenosis (MS) in the world results from rheumatic
heart disease, senile calcific MS is found with increasing frequency in the elderly
population in North America. This is due to calcification of the mitral annulus and
calcification that extends into the leaflets, which cause both a narrowing of the annulus
and a rigidity of the leaflets without commissural fusion. Mitral annular calcification
(MAC) has been associated with decreased renal function and inflammatory markers
like C-reactive protein; however, senile calcific MS is common in the elderly population
with normal renal function and is associated with senile aortic stenosis (AS).

Indications for intervention in patients with senile calcific MS are different from those for
rheumatic MS for the following reasons: first, because calcification involves the annulus
and base of the leaflets without commissural fusion, there is no role for percutaneous
mitral balloon or surgical commissurotomy; and second, the presence of severe MAC
can be quite challenging for the surgeon because it causes problems in securely
KKUH
Collected by:
Dr Hani Abdullah
Dr Fatma Hadi
Dr Salem Boresa
Dr Rami Elesali
Dr Naeif Almagal
1/1/2020
76

attaching the prosthetic valve and narrowing of the orifice. Supra-annular insertion and
other innovative techniques can be used, such as placement of a felt patch around the
valve orifice to anchor the prosthesis; however, this only works if the mitral orifice is
adequate. If the annular calcification narrows the orifice, it has to be debrided. The other
alternative is left atrial to ventricular bypass with a valved conduit in extreme cases of
calcification both of the leaflet and the annulus. Finally, patients with calcification are
often elderly and debilitated, have multiple comorbidities, and are at high risk for
surgery. For these reasons, intervention should be delayed until symptoms are severely
limiting and cannot be managed with diuresis and heart rate control.

A subset of patients have mitral inflow obstruction due to other causes, such as
congenital malformations, tumors, or other masses. Congenital MS usually takes the
form of a parachute mitral valve, where the mitral chordae are attached to a single or
dominant papillary muscle and often form a component of the Shone complex, which
can include supramitral rings, valvular or subvalvular AS, and aortic coarctation. For MS
caused by tumors or other obstructive lesions, intervention is aimed at reducing or
removing the mass, with efforts made to preserve the valve.

This patient has a history consistent with senile calcific MS as she is older and has renal
dysfunction. Thus, the image with echodensity along the posterior mitral annulus is the
best response. Answer C
Key Point
Mitral annular calcium is common, with 45% of patients >65 years of age exhibiting
evidence for this. It can occasionally lead to both mitral regurgitation and mitral stenosis.

KKUH
Collected by:
Dr Hani Abdullah
Dr Fatma Hadi
Dr Salem Boresa
Dr Rami Elesali
Dr Naeif Almagal
1/1/2020
77

Question 47
47- A 50-year-old female with a history of a murmur was referred to the valve clinic at a
large tertiary referral hospital. She denies any symptoms. Her exam is normal except for
a blowing 3/6 holosystolic murmur at the apex that radiates to the base of the heart. Her
transthoracic echocardiogram was of good quality and revealed a normal ejection
fraction (EF; 68%) with normal left ventricular (LV) dimensions and severe eccentric
anteriorly-directed mitral regurgitation (MR) secondary to posterior leaflet prolapse. Her
predicted risk of mortality with surgical mitral valve (MV) repair or replacement is <1%.

Which of the following is the most appropriate next step in the management of this
patient?
A. Cardiac magnetic resonance imaging.
B. Repeat transthoracic echocardiogram in 2 years.
C. Transcatheter MV repair.
D. Surgical MV replacement.
E. Surgical MV repair.

This patient has asymptomatic primary severe MR (stage C1) with a normal EF and
normal LV dimensions. Patients with asymptomatic severe primary MR with a surgical
risk of <1% and and a high likelihood of successful MV repair (>95%) may be
considered for surgical MV repair to prevent long-term sequela and adverse remodeling
(Class IIa). MV repair by an experienced surgeon is preferred over replacement. There
is no indication for transcather repair as the patient is at a low risk for surgery. If
observation is chosen, a repeat echocardiogram should be obtained in 6-12 months for
severe MR. As the echocardiogram was of good diagnostic quality, cardiac magnetic
resonance imaging is not indicated. Answer E
Key Point
Surgical repair rates and operative mortality rates are strongly influenced by operator
and center experience.

KKUH
Collected by:
Dr Hani Abdullah
Dr Fatma Hadi
Dr Salem Boresa
Dr Rami Elesali
Dr Naeif Almagal
1/1/2020
78

Question 48
48- A 28-year-old female who is 26 weeks pregnant presents to the emergency
department with a 2-week history of progressive dyspnea and palpitations. She has a
history of a prolonged febrile illness as a child. Her examination reveals an irregular
heart rate of 140 bpm with a blood pressure of 95/60 mm Hg. Her jugular venous
pressure is 4 cm above the sternal angle. A grade 2/4 diastolic rumble is heard at the
apex. Her lungs are clear to auscultation with no peripheral edema.
Her electrocardiogram shows atrial fibrillation (AF) at 144 bpm.
She is given intravenous metoprolol 5 mg and feels somewhat better.
Her transthoracic echocardiogram shows a left ventricular ejection fraction of 60%. The
mean mitral valve (MV) gradient is 12 mm Hg at a heart rate of 88 bpm. The calculated
MV area by the pressure half-time equation is 1.3 cm2. There is mild mitral regurgitation.
The peak pulmonary artery pressure is 50 mm Hg.
In addition to anticoagulation, which of the following is the next best step in the
management of this patient?

A. Urgent direct current cardioversion.


B. Metoprolol tartrate 12.5 mg every 6 hours.
C. Sotalol 80 mg twice daily.
D. Percutaneous balloon valvuloplasty.

This pregnant woman presents with rheumatic mitral stenosis and AF with rapid
ventricular response. The initial treatment will be medical therapy to lower her heart rate
and anticoagulation. Thus, metoprolol is the correct answer.

There is no role for cardioversion at this time, as she now has appropriate rate control
with associated improvement in symptoms. In addition, chemical or electrical
cardioversion is not indicated as she has not been previously anticoagulated and the
duration of AF is unknown; she has been symptomatic for 2 weeks.

If symptoms fail to improve with additional medical therapy, future options include
transesophageal echocardiogram-guided direct current cardioversion. Indications for
intervention (either percutaneous or surgical) are symptoms refractory to medical
management. If medical management fails, balloon valvuloplasty (with pelvic shielding)
would be the procedure of choice if the valve morphology is appropriate. Surgical MV
replacement during pregnancy carries a 30% risk of fetal loss and is therefore a last
resort. Answer B
Key Point
Percutaneous mitral balloon commissurotomy is indicated in symptomatic patients or
asymptomatic patients with pulmonary hypertension, moderate or severe stenosis, and
favorable valve morphology in the absence of left atrial thrombus or moderate to severe
mitral regurgitation.
KKUH
Collected by:
Dr Hani Abdullah
Dr Fatma Hadi
Dr Salem Boresa
Dr Rami Elesali
Dr Naeif Almagal
1/1/2020
79

Question 49
49-A 30-year-old female presents to your clinic with progressive shortness of breath.
The symptoms started 2 years ago and now occur with minimal activity. She has no
significant past medical history. She is on no medications. Her physical exam reveals an
early opening snap followed by a low frequency murmur.

A cardiac echocardiogram showed thickening of the mitral valve leaflets with doming.
The mean gradient was 5 mm Hg at a ventricular rate of 60 bpm.

What is the next best step in the management of this patient?


A. Exercise stress myocardial perfusion imaging.
B. Transesophageal echocardiography.
C. Right heart catheterization.
D. Exercise stress echocardiography.
E. Polysomnography.

This patient has symptoms and exam findings consistent with significant mitral stenosis
(MS), but her resting echocardiographic findings are suggestive of mild MS. Exercise
testing with Doppler or invasive hemodynamic assessment is recommended to evaluate
the response of the mean mitral gradient and pulmonary artery pressure in patients with
MS when there is a discrepancy between resting Doppler echocardiographic findings
and clinical symptoms or signs.

Right heart catheterization is an invasive test and should be reserved for cases of
indeterminate noninvasive test findings. Furthermore, direct measurement of left
ventricular diastolic pressure would be required to assess the mitral valve; right heart
catheterization alone would be inadequate in this case.

She does not report any daytime somnolence that point to sleep apnea, so
polysomnography would not be the next best step.

KKUH
Collected by:
Dr Hani Abdullah
Dr Fatma Hadi
Dr Salem Boresa
Dr Rami Elesali
Dr Naeif Almagal
1/1/2020
80

Stress perfusion imaging to test for coronary ischemia would not be expected to explain
her valve findings.

Transesophageal echocardiography will be required to plan therapy once a diagnosis of


severe MS is made, but will not resolve the discrepancy between her resting
echocardiogram findings and her symptoms. Answer D
Key Point
The echocardiogram is the diagnostic mainstay and is used to assess the etiology,
morphology, and severity of the mitral stenosis. The analysis of the morphology of the
mitral valve apparatus (including leaflet thickness and mobility, leaflet calcification,
subvalvular thickening, and chordal fusion) and the appearance of the commissures are
key features for the diagnosis of rheumatic MS. They also have important implications
for the choice of the most appropriate intervention.

Question 50
50- A 50-year-old male presents to your office for follow-up of aortic stenosis (AS) and
an aortic aneurysm. He is active and exercises regularly without symptoms. He takes no
medications.

On physical examination his blood pressure is 110/70 mm Hg with a heart rate of 66


bpm. The carotid contour is normal. A systolic click and grade 2/6 early-peaking systolic
ejection murmur are heard at the right upper sternal border. The aortic component of
the S2 is preserved. There is no diastolic murmur and no S3 or S4. His peripheral
examination shows no edema.
His transthoracic echocardiogram demonstrates bicuspid aortic valve (BAV) and the
following:

• Left ventricular ejection fraction of 60%


• Left ventricular end-diastolic dimension of 5.0 cm
• Ascending aorta dimension of 4.6 cm
• Aortic root diameter of 3.8 cm
• Left atrial volume index of 32 ml/m2
• Mean aortic valve gradient of 10 mm Hg
KKUH
Collected by:
Dr Hani Abdullah
Dr Fatma Hadi
Dr Salem Boresa
Dr Rami Elesali
Dr Naeif Almagal
1/1/2020
81

• Aortic valve area of 2.0 cm2

When should the next echocardiogram for surveillance be performed on this


patient?

A. 2 years.
B. 1 year.
C. 3 years.
D. 5 years.

For patients with a BAV and an ascending aortic aneurysm >4.5 cm, annual imaging of
the aneurysm is recommended (Class I). For patients with mild AS without an
ascending aortic aneurysm, serial echocardiography every 3-5 years is recommended
in the absence of a change in clinical status or physical findings. Echocardiography is
recommended annually for asymptomatic patients with severe AS and every 1-2 years
in those with moderate AS. Patients should be advised to report changes in symptoms
promptly. Answer B
Key Point
Careful monitoring with periodic clinical evaluation for symptoms and serial
echocardiogram for disease severity is essential to determine appropriate timing for
intervention.

51- A 60-year-old male with a past medical history significant for hypertension and
diabetes mellitus presents with fever, chills , malaise, and shortness of breath for 5
days. On presentation the patient was febrile with a blood pressure of 100/40 mm Hg.
An early, soft, high-pitched, decrescendo diastolic murmur was heard on his exam. His
blood cultures were positive for Streptococcus bovis. A cardiac echocardiogram
revealed a bicuspid aortic valve with severe aortic regurgitation and an aortic valve
replacement procedure is scheduled.

Prior to this patient's surgery, which of the following studies should be performed first?
A. Computed tomography of the abdomen and chest.
B. Fecal immunochemical test.
C. No further testing is needed.
D. Guaiac fecal occult blood test.
E. Colonoscopy.

Colon cancer screening is mandatory if the pathogen is Streptococcus (S.) bovis. The
association between colonic carcinoma and endocarditis was reported as early as 1951,
but it was only in 1977 that S. gallolyticus (previously S. bovis) was recognized by Klein
KKUH
Collected by:
Dr Hani Abdullah
Dr Fatma Hadi
Dr Salem Boresa
Dr Rami Elesali
Dr Naeif Almagal
1/1/2020
82

et al. as the pathogen agent specifically related to the presence of a colonic cancer.
Answer E
Key Point
Streptococcus bovis endocarditis is often associated with malignancy of the
gastrointestinal track.

Question 52
52- A 40-year-old female is noted to have a late systolic murmur on routine outpatient
follow-up. She is physically active and enjoys running and cycling. She denies any
symptoms related to physical exertion. A transthoracic echocardiogram is performed.
This demonstrates normal left ventricular (LV) size and function with an ejection fraction
of 65%. Left atrial (LA) size is normal. There is prolapse of the P2 segment of the of the
mitral valve. A proximal isovelocity surface area (PISA) radius of 1 cm is measured at
an aliasing velocity of 40 cm/sec. The maximum velocity of the regurgitant jet is 5
m/sec. The velocity time integral (VTI) of the regurgitant jet is 30 cm.

Which of the following quantitative echocardiographic features most accurately reflects


this patient's mitral regurgitation (MR) severity?
A. MR jet area.
B. Vena contracta width.
C. Regurgitant volume.
D. MR jet maximum velocity.
E. Effective regurgitant orifice area.

This question highlights the importance that systolic duration of MR plays when
assessing the severity of MR. Parameters which are measured in a single frame like
PISA, vena contracta width (VCW), or jet area can lead to significant overestimation of
the severity of MR, especially in the setting of late systolic MR. The presence of normal

KKUH
Collected by:
Dr Hani Abdullah
Dr Fatma Hadi
Dr Salem Boresa
Dr Rami Elesali
Dr Naeif Almagal
1/1/2020
83

LV and LA size should raise the possibility of overestimation by quantitative measure in


this asymptomatic patient.

When calculated using the data from the echocardiogram, the instaneous effective
regurgitant orifice area (EROA) is 0.5 cm2, which is consistent with severe MR (see
calculations below). However, the calculated regurgitant volume is in the mild range.
This is most likely secondary to late systolic MR as opposed to holosystolic MR. This is
commonly seen in mitral prolapse where there is late systolic regurgitation yielding a
small regurgitant volume. MR duration is a common reason for discrepancy between the
calculated EROA and regurgitant volume. The VCW profile of the MR regurgitant will be
helpful in defining the duration of MR.

The MR jet would be expected to be anteriorly directed in this patient with posterior
leaflet prolapse.

The regurgitant fraction cannot be calculated based on the information provided but
would be expected to be low with a regurgitant volume of 15 ml/beat.

EROA = [(2) x (3.14) x (PISA r2) x (Aliasing velocity)]/Vmax MR jet

EROA = [2 x 3.14 x 12 x 40] / 500

EROA = 0.5 cm2

Regurgitant volume is then calculated according the formula:


regurgitant volume = EROA x VTI of MR jet

regurgitant volume = 0.5 cm2 x 30 cm

regurgitant volume = 15 cm3.Answer C


Key Point
Echocardiography is the most useful diagnostic test to inform the etiology and severity
of mitral regurgitation. Echocardiographic grading of more-than-trivial mitral
regurgitation should include quantitative criteria beyond color jet dimensions, particularly
if jets are nonholosystolic or eccentric.

KKUH
Collected by:
Dr Hani Abdullah
Dr Fatma Hadi
Dr Salem Boresa
Dr Rami Elesali
Dr Naeif Almagal
1/1/2020
84

Question 53
53- A 27-year-old female who is 30 weeks pregnant presents to the emergency
department with several days of progressive dyspnea on exertion. She had a similar
presentation to the emergency department 3 weeks ago, and was given prescriptions
for furosemide 40 mg once daily and metoprolol 25 mg twice daily.

Her vital signs are a heart rate of 102 bpm, a blood pressure 100/60 mm Hg, and an
oxygen saturation 100% on 2 liters nasal cannula oxygen. Her physical examination
reveals bilateral rales two-thirds of the way up the posterior lung fields. There is an
elevated jugular venous pressure to the angle of the mandible, a 1/6 holosystolic
murmur at the apex with a 2/4 diastolic murmur, and an opening snap. There is 1+
pitting lower extremity edema.

Her electrocardiogram shows sinus tachycardia. Her echocardiogram reveals normal


left ventricular function with fusion of the mitral commissures and a mean transmitral
valve gradient of 16 mm Hg at a heart rate of 90 bpm with trivial mitral regurgitation. The
valve leaflet tips are thickened with minimal involvement of the subvalvular chords and
no significant calcification. The calculated mitral valve (MV) area by the pressure half-
time equation is 1.5 cm2. The estimated right ventricular systolic pressure is 50 mm Hg.

In addition to diuresis, which of the following is the best next step for the management
of this patient?
A. Percutaneous mitral commissurotomy.
B. Intravenous metoprolol 5 mg every 6 hours.
C. Delivery followed by MV replacement.
D. Percutaneous edge-to-edge MV repair.
E. MV replacement.

Percutaneous MV commissurotomy (valvuloplasty) is a Class IIa recommendation in the


absence of significant mitral regurgitation (MR) and a favorable Wilkins Score in the
2014 American Heart Association/American College of Cardiology valvular disease
guidelines for pregnant patients with Class III or IV heart failure symptoms despite
medical therapy. If she were not pregnant, percutaneous balloon MV commissurotomy
would be a Class I indication for severe symptomatic mitral stenosis (MS) and favorable
valve morphology.

Delivery and then surgery would not be optimal management at this point, unless for a
specific obstetric indication, due to issues related to prematurity.

Her symptoms have progressed despite diuretic and beta-blocker therapy, thus current
medical therapy would not be the best choice.

KKUH
Collected by:
Dr Hani Abdullah
Dr Fatma Hadi
Dr Salem Boresa
Dr Rami Elesali
Dr Naeif Almagal
1/1/2020
85

Percutaneous mitral edge-to-edge MV repair (MitraClip) is not indicated as the patient


has MS, not MR. Answer A
Key Point
Percutaneous mitral balloon commissurotomy is indicated in symptomatic patients or
asymptomatic patients with pulmonary hypertension, moderate or severe stenosis, and
favorable valve morphology in the absence of left atrial thrombus or moderate to severe
mitral regurgitation.

Question 54
54- A 55-year-old female presents to the emergency department with fever and
shortness of breath 3 months after mitral valve replacement with a bioprosthesis. Her
past medical history includes mitral regurgitation due to Barlow's disease, hypertension,
and hypothyroidism. Her current medications include aspirin, warfarin, lisinopril, and
levothyroxine. On physical exam her vital signs are a temperature of 101 degrees
Fahrenheit, a blood pressure of 100/64 mm Hg, a pulse of 100 bpm, and an oxygen
saturation of 90% on 6 liters of O2. She appears to be uncomfortable. Her neck veins
are flat. There are scattered rales throughout both lung fields. The precordium is
hyperdynamic with a soft systolic murmur and an S3 gallop. Her extremities are warm
without edema.

A transthoracic echocardiogram (TTE) demonstrates hyperdynamic left venticular (LV)


systolic function, a LV ejection fraction of 75%, and elevated gradients across the mitral
prosthesis.

Which of the following is the next best step in in the management of this patient?
A. Computed tomography pulmonary angiogram.
B. Cardiac catheterization.
C. Cardiac magnetic resonance imaging.
D. Cinefluoroscopy.
E. Transesophageal echocardiography.

KKUH
Collected by:
Dr Hani Abdullah
Dr Fatma Hadi
Dr Salem Boresa
Dr Rami Elesali
Dr Naeif Almagal
1/1/2020
86

This patient appears to have prosthetic valve endocarditis, possibly with a paravalvular
leak that is causing heart failure.

In patients with bioprosthetic valves who show evidence of prosthetic valve


regurgitation, TTE is used to monitor the appearance of the valve leaflets, valve
hemodynamics, LV size and systolic function, and to estimate pulmonary pressures.
The initial approach is TTE for evaluation of antegrade valve velocities and pressure
gradients. However, transesophageal echocardiography (TEE) is essential for the
evaluation of suspected or known prosthetic mitral valve regurgitation. On TTE imaging,
the left atrium is shadowed by the valve prosthesis, obscuring evidence of prosthetic
regurgitation. TEE imaging provides clear images of the left atrial side of the mitral
prosthesis and is particularly useful for delineation of the site and severity of
paravalvular regurgitation, evaluation of suitability for a percutaneous approach, and
guidance during percutaneous closure procedures.

Thus, TEE is the correct approach in this patient.

Cardiac catheterization and computed tomography pulmonary angiography would not


offer additional insight into the cause of this patient's symptoms, management, or
prosthetic valve regurgitation. While magnetic resonance imaging can be useful for
quantifying mitral regurgitation or paravalvular leaks, it would not be the first choice in
this unstable patient. Notably, cinefluoroscopy is not indicated as she has a
bioprosthetic valve. Answer E
Key Point
Doppler echocardiography (transthoracic echocardiography and transesophageal
echocardiography) is the method of choice for the diagnosis of prosthetic valve
dysfunction (stenosis and regurgitation).

KKUH
Collected by:
Dr Hani Abdullah
Dr Fatma Hadi
Dr Salem Boresa
Dr Rami Elesali
Dr Naeif Almagal
1/1/2020
87

Question 55
55- You are asked to see a 59-year-old female with aortic stenosis (AS). For the past 3
months she has noted progressive shortness of breath with daily household chores. Her
medical history is significant for hypertension. She currently takes losartan 50 mg and
aspirin 81 mg daily.

Her vital signs are a blood pressure of 124/72 mm Hg with a heart rate of 84 bpm. Her
jugular veins are not distended. Carotid upstrokes are delayed. Her lungs are clear to
auscultation. The apical impulse is sustained and displaced 2 cm to the left of the mid-
clavicular line. A grade 3/6 harsh, late-peaking systolic ejection murmur is heard
throughout the precordium, loudest at the right upper sternal border, and radiates to the
carotids. There is no diastolic murmur. The rest of her physical examination is normal.

Her laboratory studies include a hemoglobin of 13.1 g/dl and a creatinine of 0.8 mg/dl.
Her echocardiogram demonstrates concentric left ventricular hypertrophy with an
ejection fraction of 65%. There is a bicuspid aortic valve with AS; peak velocity is 4.2
m/sec with a mean gradient of 42 mm Hg and a calculated valve area of 0.7 cm 2.
Coronary angiography reveals a 60% angiographic stenosis of the mid left anterior
descending (LAD) artery; fractional flow reserve (FFR) was 0.89. Her estimated
operative mortality for cardiac surgery is calculated at 1.6%.

Which of the following is the next best step in the management of this patient?
A. Surgical aortic valve replacement plus coronary artery bypass grafting.
B. Transcatheter aortic valve replacement plus percutaneous coronary intervention.
C. Surgical aortic valve replacement.
D. Transcatheter aortic valve replacement.

The patient has symptomatic, severe AS and nonobstructive coronary artery disease
involving her LAD artery based on an FFR of >0.8. Her surgical risk is low (<4%). The
most appropriate therapy is surgical aortic valve replacement (AVR; Class I) (Figure 1).
Transcather AVR is a Class I indication for treating AS in patients at prohibitive surgical
risk as well as high surgical risk (Society of Thoracic Surgeons Predicted Risk of
Mortality score >8%), depending on patient values and preferences. Transcatheter AVR
is not currently recommended for the treatment of patients with AS and low surgical risk.
She needs medical therapy for her coronary disease. Answer C

KKUH
Collected by:
Dr Hani Abdullah
Dr Fatma Hadi
Dr Salem Boresa
Dr Rami Elesali
Dr Naeif Almagal
1/1/2020
88

Question 56
56- A 63-year-old male with history of ST-segment elevation myocardial infarction (MI)
of the right coronary artery presents to the cardiology clinic with progressive exertional
dyspnea. He denies any chest discomfort with activity. He has not been seen for 18
months. His medications include aspirin 81 mg daily, metoprolol tartrate 25 mg twice a
day, and lisinopril 2.5 mg daily. On exam, his blood pressure is 122/78 mm Hg, heart
rate is 95 beats per minute, respiratory rate is 18 breaths/min, and oxygen saturation is
93% while breathing ambient air. His examination reveals a III/VI blowing holosystolic
murmur loudest at the apex and radiating to the axilla. He has trace peripheral edema.
His electrocardiogram shows sinus rhythm, inferior Q waves, and a left bundle branch
KKUH
Collected by:
Dr Hani Abdullah
Dr Fatma Hadi
Dr Salem Boresa
Dr Rami Elesali
Dr Naeif Almagal
1/1/2020
89

block (LBBB) with a QRS duration of 143 ms. Echocardiography reveals severe inferior
and inferolateral left ventricular (LV) hypokinesis, a LV ejection fraction of 34%, and
severe mitral regurgitation (MR).

Which of the following is the next best step in the management of this patient?
A. Optimization of medical therapy.
B. Cardiac resynchronization therapy.
C. Mitral valve annuloplasty with repair.
D. Mitral clip placement.
E. Mitral valve replacement.

MR may be primary (due to intrinsic valve dysfunction) or secondary (due to adverse LV


remodeling). This patient presents with chronic symptomatic heart failure (HF) in the
setting of a prior MI, likely ischemic cardiomyopathy, and most likely secondary MR due
to papillary muscle displacement with posterior mitral valve (MV) leaflet tethering. For
patients with secondary MR, treatment is initially aimed at improving ventricular function
and volume status.

This patient is on metoprolol tartrate rather than metoprolol succinate and is on a low-
dose of an angiotensin-converting enzyme inhibitor, so there is still opportunity to
optimize guideline-directed medical therapy (GDMT) for HF. If he had persistent LV
systolic dysfunction and HF symptoms after optimization of GDMT, then cardiac
resynchronization therapy may be considered given his LBBB and prolonged QRS
duration. For secondary MR, MV replacement and repair (Class IIb) were shown to
have similar mortality and LV remodeling outcomes in a randomized trial, but MV
replacement was associated with a lower rate of recurrent moderate or severe MR and
lower incidence of both HF and repeat hospitalization. Both surgical groups had a high
mortality rate at 2 years, emphasizing the poor prognosis with secondary MR.

Neither of the surgical MV treatments nor transcatheter MV repair would be a first-line


therapy for this patient who is receiving suboptimal medical management. Answer A
Key Point
Causes of mitral regurgitation may be broadly differentiated as primary (predominantly
degenerative) or secondary (functional), a classification that has significant impact on
prognosis and management.

KKUH
Collected by:
Dr Hani Abdullah
Dr Fatma Hadi
Dr Salem Boresa
Dr Rami Elesali
Dr Naeif Almagal
1/1/2020
90

Question 57
57- A 74-year-old female with a history of hypertension and moderate aortic stenosis
(AS) presents to your outpatient clinic for a routine follow-up evaluation. She reports
worsening fatigue and dyspnea, but no chest pain, palpitations, presyncope, or
syncope. Her blood pressure is 112/80 mm Hg and heart rate is 66 bpm. Her last
echocardiogram, 1 year prior, demonstrated normal biventricular function, mild mitral
regurgitation, and aortic peak velocity 3.1 m/sec with a calculated aortic valve area 1.3
cm2.

What physical examination finding would suggest progression of this patient's aortic
stenosis?
A. Increased opening snap-S2 interval.
B. Ejection click.
C. Widened splitting of S2.
D. Enhanced A2.
E. Late-peaking murmur.

Aortic valve stenosis is associated with reduced leaflet mobility and delayed closure.
Auscultatory findings in severe AS include a soft, single S2 sound since the
A2 component of the second heart sound, which is due to aortic valve closure, is
delayed and tends to occur simultaneously with the pulmonic component (P 2), which is
due to pulmonic valve closure. The S2 may become paradoxically split when the
stenosis is severe and associated with left venricular dysfunction. With increasingly
severe, fixed AS, the A2 closing sound may disappear. The presence of a normally split
S2 is the most reliable finding to exclude severe AS in adults.

The first heart sound (S1) is usually normal. However, an aortic ejection click, which is
more commonly heard with a congenital bicuspid valve, may be heard after S 1 early in
AS when the leaflets are stiff but still somewhat compliant and mobile. Vigorous left
atrial contraction against a stiff, noncompliant ventricle can produce an S4.

The murmur associated with AS is described as a systolic "ejection" murmur, typically


heard best at the base of the heart in the right intercostal space, with a harsh quality.
The murmur may be heard at the apex especially in elderly patients with radiation of the
high frequency sounds (Gallavardin phenomenon). The murmur generally begins after
KKUH
Collected by:
Dr Hani Abdullah
Dr Fatma Hadi
Dr Salem Boresa
Dr Rami Elesali
Dr Naeif Almagal
1/1/2020
91

S1 and ends before S2. The intensity of the murmur reflects the amount and velocity of
blood flow across the valve and the turbulence produced by the stenosis. A loud
murmur (grade 4 or greater) has a high specificity for severe AS. However, most
patients with severe stenosis have a grade 3 murmur, and many have only a grade 1 or
2 murmur. In patients with low flow, low gradient AS the murmur may be soft and almost
inaudible.

The timing of the murmur also correlates with the severity of the stenosis, similar to the
timing of the carotid pulse. An early-peaking murmur is typical for mild to moderate AS,
while a late-peaking murmur is consistent with severe AS.

The opening snap is a finding of mitral stenosis and not pertinent to AS. Answer E
Key Point
As the murmur of aortic stenosis (AS) worsens, the ejection sound and the intensity of
A2 diminish; the murmur peaks later in systole. The AS murmur increases after a
premature ventricular contraction.

Question 58
58- A 45-year-old female with mitral valve prolapse (MVP) for many years presents to
your clinic for re-evaluation. She feels well. She teaches a spinning class at her gym.
She takes no medications.

On exam her blood pressure is 100/60 mm Hg and pulse is 62 bpm. She has a mid
systolic click followed by a loud murmur radiating to the axilla.

Her echocardiogram demonstrates a left ventricular end systolic diameter (LVESD) of


4.1 cm and left ventricular ejection fraction (LVEF) of 60%. There is posterior MVP
resulting in severe mitral regurgitation (MR) with a calculated effective regurgitant orifice
area of 0.45 cm2 and a regurgitant volume of 70 ml.

KKUH
Collected by:
Dr Hani Abdullah
Dr Fatma Hadi
Dr Salem Boresa
Dr Rami Elesali
Dr Naeif Almagal
1/1/2020
92

Which of the following is the best strategy for managing this patient's mitral valve
disease?
A. Bioprosthetic valve replacement.
B. Mechanical valve replacement.
C. Surgical valve repair.
D. Watchful waiting.
E. Transcatheter valve repair.

Mitral valve (MV) surgery is recommended for asymptomatic patients with chronic
severe primary MR and left ventricular (LV) dysfunction (LVEF of 30-60% and/or
LVESD of 40 mm, stage C2).

The goal of therapy in MR is to correct it before the onset of LV systolic dysfunction and
the subsequent adverse effect on patient outcomes. Ideally, MV surgery should be
performed when the patient's LV approaches, but has not yet reached the parameters
that indicate systolic dysfunction (LVEF of 60% or LVESD of 40 mm). Because
symptoms do not always coincide with LV dysfunction, imaging surveillance is used to
plan surgery before severe dysfunction has occurred. If moderate LV dysfunction is
already present, prognosis is reduced following MV operation. Thus, further delay (even
though symptoms are absent) will lead to greater LV dysfunction and a still worse
prognosis. Because the loading conditions in MR allow continued late ejection into a
lower-impedance left atrium, a higher cutoff for “normal” LVEF is used in MR than in
other types of heart disease. Although it is clearly inadvisable to allow patients' LV
function to deteriorate beyond the benchmarks of an LVEF of 60% and/or LVESD of 40
mm, some recovery of LV function can still occur even if these thresholds have been
crossed.

MV repair is recommended in preference to MV replacement when surgical treatment is


indicated for patients with chronic severe primary MR involving the anterior leaflet or
both leaflets when a successful and durable repair can be accomplished, thus MV repair
is the preferred choice in this patient.

Transcatheter MV repair may be considered for severely symptomatic patients (New


York Heart Association [NYHA] class III-IV) with chronic severe primary MR (stage D)
who have favorable anatomy for the repair procedure and a reasonable life expectancy,
but who have a prohibitive surgical risk because of severe comorbidities and remain
severely symptomatic despite optimal guideline directed medical therapy (GDMT) for
heart failure (HF; Level of Evidence B). A randomized controlled trial of percutaneous
MV repair using the MitraClip device versus surgical MV repair was conducted in the
United States. The clip was found to be safe, but less effective than surgical repair
because residual MR was more prevalent in the percutaneous group. However, the clip
KKUH
Collected by:
Dr Hani Abdullah
Dr Fatma Hadi
Dr Salem Boresa
Dr Rami Elesali
Dr Naeif Almagal
1/1/2020
93

reduced the severity of MR, improved symptoms, and led to reverse LV remodeling.
Percutaneous MV repair should only be considered for patients with chronic primary MR
who remain severely symptomatic with NYHA class III-IV HF symptoms despite optimal
GDMT for HF and who are considered inoperable.

Degenerative MV disease consisting of more than posterior leaflet disease requires a


more complex and extensive repair. When the anterior leaflet or both leaflets require
repair, durability of the repair is less certain, with a freedom from reoperation of
approximately 80% and a freedom from recurrent moderate or severe MR of 60% at 15-
20 years. These results are superior to the results of MV replacement, even in elderly
patients. Repair should also be attempted if possible with other causes of severe MR,
such as papillary muscle rupture, infective endocarditis, and cleft MV. As the repair
becomes more complex however, results of very complex repair in younger patients
may be matched by results of durable mechanical MV replacement with careful
management of anticoagulation. Answer C
Key Point
For primary mitral regurgitation, surgical mitral valve (MV) repair is recommended over
MV replacement because of preserved left ventricular (LV) function, lower operative
mortality rate, and lower rate of complications associated with prosthetic valves in the
long term.

Question 59
59- A 65-year-old male calls your office with questions regarding his mitral valve
prosthesis (MVP). Two years ago, the patient underwent surgery with placement of a 31
mm porcine MVP for symptomatic, severe regurgitation due to myxomatous disease. He
has done well since surgery, with no symptoms or complications in follow-up. His last
echocardiogram, which was performed 2 months ago, demonstrated a mean transmitral
gradient of 2 mm Hg and trivial mitral regurgitation.

For which of the following procedures should he take antibiotics as prophylaxis against
infective endocarditis?
KKUH
Collected by:
Dr Hani Abdullah
Dr Fatma Hadi
Dr Salem Boresa
Dr Rami Elesali
Dr Naeif Almagal
1/1/2020
94

A. Fat pad biopsy.


B. Esophagogastroduodenoscopy.
C. Cystoscopy with urethral dilation.
D. Dental cleaning.
E. Colonoscopy with biopsy.

Antibiotic prophylaxis against endocarditis is indicated for those patients undergoing


dental procedures involving manipulation of gingival tissues who are at highest risk of
complications. These high-risk patients include those with a prosthetic cardiac valve or
prosthetic valve repair material, a prior history of infective endocarditis, cardiac
transplant with valvulopathy, completely repaired congenital heart disease (CHD) with
percutaneous or surgical repair occurring within the previous 6 months, repaired CHD
with residual shunts or defects that impair endothelialization of prosthetic material, and
unrepaired cyanotic CHD.

Antibiotic prophylaxis is required in these patients who are undergoing dental


procedures that involve manipulation of the gingival tissues, the periapical region of the
teeth, or perforation of oral mucosa. There are no prospective studies to suggest benefit
from antibiotic prophylaxis in nondental procedures. Answer D .

KKUH
Collected by:
Dr Hani Abdullah
Dr Fatma Hadi
Dr Salem Boresa
Dr Rami Elesali
Dr Naeif Almagal
1/1/2020

You might also like